You are on page 1of 512

Table of Corrterrts

l~Diffcrntial Calcufus Preliminaries



Rolle's Theor .m

Lagrange's MeaI1. Value Theorem Cauchy's Mean Value Theorem Taylor's Theorem

M I ,., ~

ac aurm s expansion

In detsrmdnatc forms

o DO

Limi ts of the form 0 1 DC

Limits of the fOT"m 0°, ODO~, 1 w Deri vative IOf are length Cartesian curves

POhlT curves

Curvature

Radius of curvature in Cartesian form Radius of curvature in parametric fbrm Radius of curvature in pedal form Radius of curvature in polar form Centre of curvature

Evolutos

Taylor's Theorem fOT {(x .Y} Maxima and NIlnin13 fOI~ f(x, J~) Method of undetcrmirred multipliers

2'~ Irrteg ra ' Calculus'

Double Intcgruls

Chu ng-e of urdcr of integration (~'~lan~p nr va riablcs

GP.l nnr-t ric al a p'p lica t i, rms

. . ..

~ I! !I

II iii Ii

!t i Ii

, " .

III! • •

• i' 11'1

rF. ill •

· ~ ..

III! • •

~ iii •

•• III! •

• e- ..

, . -

• m •

1'~13:,51 1 3 10 18 2:2 ~ "17

48

S5

f5'9 64 65 70

74 75 81 ss 87 '91 96

100 12 125

- 3':-'6 2'-'3-'6',

iii llil - .. 1, .!!!!!!!l .. - .•. 1

.. . ~

36 14,8 1 F} !<-)' 171

Triplv Integrals Changf..l of variab'~f~s (~0111 p utatinn of vo lume

(jHJnn13 and BetL1_ functions 'l'l 1 e (~H Jl1 m a fin ret i 011

The Beta tunetiun

Helutinn botwoon Bun .. n) and Illl)

3, Differ-ent.ia] Equations Linear 01] fferenti Ell equations

· . .

185 192 200

'2014 204 217

211; F-: .~j)

· . .

Ii i ill

, .. .

237

Hunu )gell(~t'UM cq uations of second-order . . . 237

NOI1-·1101110geIleuus equations of sr cond-ordor . 243,

/

Inverse differential operatur

P ~,. m • t I 1 X.» k .. en'.\"

arrrcurar in .8g'r'3 W If'n -

Particular integral when

X = h. gin 'ret·1 b) Or' h COR (ax"~ b)

Particu lur ~nlrgta~ w hen J.Y is a polvnomial

Part i ~:1l1 a r intogr: rl when X = efLl' V Eq,ll.atj'o·ns of third and higher orders Homogeneous li near cq nations

I\·fet]10d, of undet irrnined eoeffici ants Method of variation of par ameters

4" Laplace Tr-ansforms The j .aplaee Transform

,

'I'ransforms of ~()nl(' common functions

t Transform of til (n. f f(f) lit

o

'I'run ~forn) of a periodic function 'I'runsf irm uf t lu- ~St.p~ II lu n t;t i( Ul

Tr a n ~ Cur m of t ht ~ Im p LI L~{-i fu n ctiun

The i( ;0 nvol ~J tj~)m ~ theore-m

, ..

245

· . -

· . ~

'256, 263

269

280 295 ,at]7

:3.20

• II •

~ 'i 3;14-429 · ~ . ;)34

• ! ...

34 I

· ..

, . .

;1Y,S

~JlI".! "J ·0]110,..;....

;]70

3;7;1

1'11 Inverse Laplace Trunsform

Evaluati "L 1 L~ I

nva ua Jon I 1~I(S - ~)I

Evaluation of L 11 i e - (IS Fl.'!) :

Evaluat.iun If L 1: F(:; lis I. L -1 [ r» liil :

I; valuat.ion )L' L 1, I //(S): by cun\ ~'U~ tun' h U~"( m Laplace Transform method [or Dif~rentia]

equ a tions .""

orn ,A ppIJmC'a'~ i'Ol1S Vibrations of a spring Deflection uf beams LRC circuits

5~ Vctor Calculus

Orthogonal curvilinear coor diria ~es I' artieular eases

Gradj8nt~ Divergence, 1 a ' h~ cit 'a d Curl '4'x.prt~~Slon for, {p

• xpress i on for div f

E ~ t" \,". J~ ',i1

X press ~on ~ 0. r y \t~

Expn 'SS ~(, I, fi rr C'U rl f

Intearation of v cl 01." fun .tion: OJ Line integral

Surface in egra]

Vulun~e irr I ~,gr~::, ~

111tugrnl 'Th eorerus

Green TS Th lOT"llnl in the plane St k 1 frl

1 :0 l~S ~~ , n( ~url \nl

Syllabus

M,' ,d' I Ques,ti'oD Pp . rs E,x:amii'natiJon Q, eSltion. P,apers lnd x

" - ,

:i~l 387 388 ::19:3

~ . ..

~'396 408, '-l08

'17 42;1

· "" ,

· . .

.' I~ 43(-5,08

· . . 4301 437

,447

· . .

· ..

463 '6a 46~ 472

" ,

40 • oh

490

.. II! 50'9-51.
' . .. 511 .. 516
Iii Ii 517-540
III ,m 541 .. 54~ Chapter- 1

Diffeeerrtinl Calculus

This chapter is a continuation of Chapterz of Part 1 of the book, In this chapter, \~.re first p.ruve some fundamental theorems, known as Mean. Value Theorems; and some of 1..11eiT applications. Then, we take up the topics of de rioa tioes .of orr and curuaiure of plane curves" Finally, we consider tile topics of expansions of

[unctions and lvlaxinLu and, Minima, in respect of tunc ions of several variables.

'V*,i begin with a Sill m mary of som 8 basic {[cfinitiun~ and results already known to the reader.

(1) Let a and h be t\VO real numbers wit li {( < h. Then "the set of all real numbers Jt such that (l < ~t"' < b is call ld thr- elos td in terual la, b]. Gl1olllPtrically" la~, b I represents the line segment un the .r-axis. having x -= tI and .1f ~ h as end points, (T]lC enrl points are also supposed to belong to this line seamen t),

(2) Give-n twu rca] numbers a and b with o < b . tho set of ,all real n umbers J[ S uch tll,Cl t n < Jr < b is called thr- (J1Jen in teroal '~a; b,. Geometric» 11}r,; ((f, b) rc -pres (:;~n ts the li ne SPg men t on t he .r-axis that lies !:rtr,~,IL:tl~\f betueen ~h.e points _t '= ,0 Hll,d x ~ /J. cr 11 t' POil1tS x = a and x ~ b aru not s'upposed to lH~]Ullg to this nnfl segrn ent i.

(3) If (\ functiun 11xJ iH defined for all values nfr which beIr~J;).g to an intervn] .I~, we say that Ilx} is d(:f~n0d in lh,~ irrt.~ -rval 1 ~ The intor'valmay be a closed intcrvul Of a n upon interval.

(4.) Let 1 be an ·i ntervnl 311d a be u point f rp(Jlln,lLulliJC.·) belnnhT),]Jg- to this intervul. Also, let lLl-) h(·~ H, function defined in I. ",1 e say t but Il.r 1 is con t- in U{)II.',~' at the: poin t JP = '"' if

If fL t'" is CUll i in uo us al l rv ~r.v' point or 1, V~lP ~ ay that II Ilx) j ~

t' .. I II

tOll U1UI ~U,S LH ' .

(iC"Onlllirically. 11/( v) is cont.inuuus at .X == a II J11eanS th at the gr,.1ph [jf I{x} hns nu jump at tile puint _r ~ a . Also, '!f(x)]6 conl inuous in an interval I II means that t1H\ graJ)h of t(x) has no jump .uiywlu-re in the interval J .

For example, consider the function t(X) defined in thp interval I ~ 1., ] J by

Ilx) r - 1 for - 1 < JC < 0
- 1 for 1
~\:" 0< x< TI1~ ~Tapb of this fLL11Ciiol1 is SllU\VIl in Figurp L'L TIle gra~ph indicates that the function is diecontinnous at the point X-= O . ... At (\v{~'r~~ other poi nt or tl1{.i ill terva l , the funetion is continuous.

- 1

-'----I, - 1

Figure 1.1

(5) Lot ft.).'} be defined inn n in terv a] I an d a bEl' a point of the int - rval. \Vt~ ~ay thai- f~:x) it-; diiicrcntiabl» a/ the point o if the dl~riva t ivr- l t(J;) exists at x - a: that is

~ : l (.11: + Ii) - f" (j ,~ I~' i! ,

lJill I == .. (u)

/ n I

J ---i- ~ ~

pxi~.1 s.

If t{ x) .j ,~ d i ffe r p n t i a b] e LL l (l v r·1 r,Y po 1 n t of 1, w p .~ a v t h D i 'I / t x) i t1 eli Jl<-lrpl1t.i u1.dt~ ill 1 ".

1. Difr(~rt_'iJ I ia J Calculu»

"

oJ

(~ tri I I II I'i . ] - r'r l' I 1 . I I

J(-'Ol11L~ ... ricai y, j{x) It'i ru ieren ran f~ at a point .'11:- = a rn(~cHl~

that 'the graph of t(y.l has a uni q n(~ t angen L at the point x == a - Also, ~r I(Xl if; dilfcrentiablc ill all irttc-rvnl /11 means lhat thp f~Taph of t{x) over t.he interval J has a uniquo tangQnt. at every uno of iL~ points: in other words. l]le g-rapll of !(.J:) over I is a smooth curoe - a ell rve wi t h no "corner pOI nt.s''.

For example, consider the function It,:¥:.) defined in the int.urval I - 1; 1] b .Y / (X) = I.x I ; J_ll at i ~ ,

l(x) = { -;

fat - 1 < .x < 0 for 0 < x < ]

TI1P grapll of th is funct.ion is 8J10\\"]1 ill 14~ig-urc 1.2. 'This functiun is not differentiable at the pointx ~ n _ (Observe that the origin is a corner point where tile tang'('Jlt i~ not unique). ]1'01' all other values ofx in the inte-rval. the funct.ion is diffcrr-ntiablo.

o

Figure 1,2

(6) The following is an important result:

"I]' a [unction /(r) i,~ differentiable in an interval. it t« (auiomnticoliy) contin uou s in thul intervul . II But tbt- converse i~ not necessarily true: that is, if' (J [unction is ronlin uou« in on in tIJrV{I!, i I need not be (I iffercntiublc in tho t internal. Ob.~E:~rve that the function ShtHV J1 ill Fiau r(, 1.2 is contin UOitS but not ditferenti able in t hc- inte-rval [ - 1; 1] ,

'I' be ft) 11 0\,\.' in g theorem. know n as t hv Ro! {. ~ 's TI icore n l 1 n: U11 cd afte-r tho F'rcn ch muthcrn ut ici [HI Mic he ,U r-<ul'h·· j IG!)~-l t179, I. i~ ()n(l ul' the fund arnr-nlul theore-ms ~~" Oiflf'~·(·!.lt inl Culculu».

4

T.l1. En gine..el""il1.g Mel t bematies-H

Theorem .... Suppos« a [unction t~x) satisfie» the follou intI three con tliti on s:

(I) "xJ is con tin uous in a closed in teroal la, bJ . (2) {(.x) is diller inliable in th e open in terual (0, b) ~

(3, {(a) = f(b,.

TILen there exists at. least one point c in the open interval (a., b)

~

such. that t·, (e) == 0 .

Geometrioal Mean.irlg

TIle Rolle's theorem has an important geometrical significance.

This significance is described by restating the theorem ill geometrical terms, as follows:

Suppose a curve C ill the xv-plane having A and B as end points satisfies the fbHo\ving conditions:

(1, The curve is contimrous at the points A and B and at every point. between A 811d B .

(2) The curve is smooth at every point between A and B .

{3,) The points A and B are at the same perpendicular distance from the x-axis,

Then th ere exi s ts at least one point P on tll.e curve (between 44 and B') at which the tangent is parallel to the x-axis.

In the above description, the points _f\ Band P correspond to x == Q~ X == band ,:r = c reepectively. See Figure 1.3.

::l .. ,

p

I I

: {(aJ

I ~

I I I I I

I

-~-- t"'----~~.--

I

I I J I

I

~~------~~~'_--------~-----ax

o a c b

Fi.~i' rc 1.;3

1. Differential CClleuius

5

The proof of the Rolle's theorem is obvious from the geo111ctrical form of thn tIleOr(~nl. The analytical proof' is 'beyond the scope of

this t.ext.

Rema "ks

(1) Ev(_~ty 011(" or the three conditions stHV(~d in the Rolle's theorem is necessary fo: the then rem to be va li cL The fllll.owing three figures illustrate the situations where one of the conditions is. violated and corisequerrtlv t.he theorem iH not valid.

In Figure 1.4(a)~ the CUI~le is discontinuou 8, Tn If] gure 1. 4( b), the curve is not smooth. In Figure 1.4fc)~ f (a) 7: f (b). In these "cases, there is no point on the curve where the tangent is parullcl to the x-axis .

. Y

~~----~---=~~~------' x

I (i' ) f7b):
I fa
I !
0 a b .Y

-

: f(a)

I

o

a,

x

Figure 1.4(a)

Figure 1.4(b)

y'

I
I
I
feb} :
. : fCa) I
I
I ' I
X
0 a b (2) When all the three conditions of the Rolle's t.heozem arc snt.isfied, t.h :.re uiav exist mnre than one puint on the curve { L. which the tangent is parallel to the .r-uxis; .sec Flgur~ 1.5. 'I'hu theorem gllHrHulees the existence of at IC(lL~'1 onc~u(:11 point

ti

y

p

-J(----

i

r( u ) :

I

~ ~

- • /~·l) Q I. (J .

- J

--+-----~----------~------------~x

b

()

u

f4~xa'n)plp l , Yeri]» iuhctlier flu' Rolle'« theorem. L~ ool id [or the

'I •• L) •

I U n C t ton 1(,1"") == "x"-' I JJ t h r t n ie n. a I r - 1; 1 I. f l H u~ fr n d ,_. such tho t

t' u·) =: 0 .

» flpre ley) == x~ . Th is fu nct.iun i~ cnntiriuous and di tl'orpl1tiahlp for all .r . Tl1erel'nrp,. if is coritirnums in l - 1~ 1 i and differenti able in ( - 1, 1) - Also" t ( _ 1) == 1 and r (I ~ = 1; that is .. the values of the function at tl1t~ ond poiut H or thp irrterval ~ _ 1 ~ ] J are equal,

Thus, It_\:' ,) == x~ satisfies all the three conditions of tho Rolle's theorem in [_. 1,~ 1]. Ht-ncc, thUft! 11lUSt. ,exi,~L a real number c between ~ 1 and 1 xuch th at l' (€') == o. Since r 'ty):::: 2x,. we verity that c == a 111 { "pt~ the cond itinn f'; (c} = 0 with -] < c < J .

Thus, fur the g_lvl~n funct.ion , -th(~ f{olh-\;~ t heorern is valid, and

l;' (r: l:::: 0 r {U- c ~ (). •

Exalnple,2 ... Shou: that the l-?ul"<~' th eorrm iH not t-'(J/t·d [or the

[uru-tiun !(-") I x I in lln: inleruul 1-1~ u .

» Tin.' gr::lph of rh« uiven function, ILr~ == j x I i~ shown jn

Fig'ure l.,~. Frrun tJH_~ Fuiurt w« lir~_d thut !(_r} is cont.irmous ill f - 1, I] arid ;-, - 1) - j"( t) == 1, but l~ not. di n(~rt~LLtiHld(_· in ( 1, I) _ T'h us, Ilr) == I:r I sui.isfics ~ mlv l Vv'O u r t.lu- thrpp cnnditions of tlle I-{nJh~'~ th('orl'ITl in [ 1 t 11. Thl)r~~f~H~P~ 'j hrRulle's t.huorvru i~ not vul irl h~TP.

1~ DilTerentinl Cal('ulux

7

Example ::l~ Verir,~ th (J! Holte '8 tlieorcn, lor tlu! ill! l()ul£ng [u» ction s:

(i) Il_r) = ( sin X)/p-Ir in 10, IT1

( 1• ~II· J I' , III

f_l) = lJ: - (},) (x

. .

eo~ x - SIn x

~J,':

which pxi:d_~ fOT all x (finite), Thus, f~x) is diflcrent.iable anrl therefore continuous for all w . In particulal", ILl{) is continuous ill [0" reJ nnd d ifferen t.ia ble in (0, 7l). Also,

sin 0

~ ----= U

.;. .

i'

.

sin IT l r IT) -

"rr' , ..

{'

- 0

su t hat r ( 0) == f (" rt) .

Thus. tl:x) satisfies all the three cunclil.iuns of the Rolle's t h aorum.

N o VI/ , f' J (L\:) = 0 rl!q u j res cos x - ~i n J_~ == a ~ which is satis f ed by x -n14 in. the irrtervnl {O, 7!). That is .. there is a (_"~ uarnely rc/ 4 ~ such that t '(c) == 0 and 0< c « rr.

Thus, for the gi vcn function. the Rolle's theorem is veri fled.

(Ii) For the 6r1v(-!n /(:r), \V I f nd til at f '(xl - 1; eX sin .r, which exi ~t~ lor :j U .r . Thus, J~x) Is different La hlc and therefore co 11 tinu U U S fo r a 11 :r. .;~_I.~ 0 ~

1'[ ; ) = t ( ~JIT ) -

I r:: 5 1

(Gr: .... n' - ,nI' ,_ 1t _ j \

r-: , SlH -- - C(~t-i. .,

- 1 4

'I'hus , t.lu- given t{x} sat .sfies all tJ1l.! th.rtcc condition ~ oi" the R~ t] le "J~ theurem in r rc/4, f1 IT 141 .

8

Now, l) (x) == 0 requires (~",. sin, x == 0, TIlls is satisfied by x·== 1[' in the interval (rr:/4 ~ R;rr/4} ,_ That is, l.hera is a c, namely 1t , such that f ~ (c) == 0 and :rc/4 < c < 51[/4,

TJ1US, for the given function ~ the Rolle's theorem is verified. ( iii) For tho gi veil ftx) ~ we. find that

/" nt~l_ bn-1 1,

(x) = (x- a) (x-) r (In +It)X- tn10+n.a)

which exisl.s for all .r . Thus, 11x) is differentiable and l11Ptefure con tin llOUS for all x , Also ~ ll[ u ) = 0 = f' (b) ~ Thus j ft~-r) s atisfies a][ the three conditions of the Rolle's Lheor en ... in raj b] "

Now, f' (x) == 0 requires (In 4~ n).x ~ (ntb + no) == 0, Hence, if we take

nib+ na

c-

nl + n

then f' (c) = O. Also, c di vides tho line segment joining the points x == a and x == b internal ly in the ratio 111 : n ~ Hence a<c<b~

Thug, fo·,. the given function, the Rolle's theorem is veri fied. •

Exampfe 4.. If' c is 0, rea. I constant, show that the equation. .x8 - lax + c == 0 cannot Iuiue tioo distinct roots in the in terual [O~ 4J ~

» Let ttx):::: _1:.3 - 12x + c , TIl ell A x) is contin uous and differen tiable for all .. x .

Su-ppose f(x) == 0 b as two distinct roots a; and b , where a < a < b < 4, Then f (a) ::- a = t ( b) so thnt f(~lf_) .111 eats all the

· three eonditions of the Rullo's theorem ill [0: I b I ' Therefore, there exists a c in (a-~ b) such that f I (c) == 0,

Since f 'Cx) == 3x2 - l2x == 3x (x - 4), f" (C) == a holds for

c == 0 and (! == 4., But both of t1108c care outside the np~n

interval (aj, b), This is a con.tr adiction.

Therefore! f(x} == a C£l11110t have two rlisfinct rootR in [O, ~t 1, •

Ex amp I e 5.. If (I 0 J U 1 ~ n 2'~ · . a n arc re 0' L !ij_~t U I S to u," s ,S ~ sc h tho].

u~ n - 1

(I. 1

.. -i~ _11-

2

U

JI

1

o

. · · (i)

'~ II 1

a nX + l1 1 X ~. • - . + a x (t

u 1.1-1 ,n

o

( ..

. .. 'Ii)

hn.<:. ot lea .. st. one reol ruot in, ~/U!. interval.' (O,~, 1)., » Cons ider he function

f1: n

Un" I + 1 n

-x X+'"

n + ] n

. . . (iii)

It ,in -oe ":'r) is a poi >nrul1lat it . s ditfcrvr tiable ~ m d 111rL~r fort eontinuous for all ~'C, A~~o, '\\~'C [lute that

. ~ . (iv)

Further,

flO) = 0

/1 } -=-

n o

Thus, ltx) satisfies all the crmdrtions of the Ro lle's hcorem in the interval lO~ I]. ancc, there is at: such that f'(·) == O. with o c c < 1. That lSI' .he p(~uat.~n~l f fIUt~) - 0 ha~ a real runt II n thp in orval (Il, 1). Stnel {'.1: == 0 is indeed tJte given 'IQl1K liun ii

[by vir u .' of expression (ivl] , the re uirerl r tilt is proved. •

I~ Vc fwlr:V . hplRuU(lts t.h-~tl~~(!Ul for IJu.~ oliu\\,"in,g Iuncl.icns. In .lach 'a,~Cm fIud an nppropriut: (. such ha {'I('J ~ () .

) .')

(i) Itx) -= x' ~ ,:3:r~ L~~ + ,:~ in l I, 3]

(i v ) II x) = -'" (.r r :1) ~ t

-x . r

- UlJ •

r 0

i v ) ILl') - xix' - (J) ,e j, n I f J, 11) ~ a > U

~ vi} It 'i,:} - 1 ng [{x~ + (J b ~I tu j h' x lin 1([, b L !J~" u > 0

'2 ~ lf (" -I ~ ::1 ]_'t!H 1 ('nn?!tan l. t-; how t1 ~.a t the" ~Iq ual.io n x:1.+ ax + t· - 0 cunnnl h; I V[~ m ure l han on€:- rnul root.

:1. H .\J t'j n isi during l h c n .. u u:tiun Ax J =- t.,· ~ a ,I ug x, (:I, > ] ~ 1i how that

I. h e ~~q 11: I U on 41[ J O,~ J' ~ (l. .r has aU ~a~ tune real root in the i utcrval

~.~~ o)_

4~ Rhnw Lha't the equal Ilun [,H,ll, J.- -= [ Ji' ha~ a Y.C:.l~ ro~'t in the iutc:r"'\nd

~ n. [,

An~'wer,s

1" (i) (. -= i f :21 ,1.3 ~ i i) to ~ - II 2/:~

(j i i) f' = rt I ~.ll :~.bLl 2 (vw l (~= ,'nb

1.4 Lagr-ange's Mean Value Theorcr.n

Bclow wa provc a Iundomentul t heorem known .a~ the First Mean "(flue Theorem of" Diffl~Ten,tiul Calculu«. r'1rl'~i~ theorum i~ also called the Lagrange'« Mean Yalno Theorem in honor of its inventor J.L. Lagrange (1736-1 ~ 13)J an ItaJian-l'fr-cn.ch mnt hemat.ician ofgreat repute.

l~ltPore'IJJ: i.9upjJosr a {unction- l?x} satisfies the [olloioinu tioo ~~ con d itions:

(1) Itx) is continuoa.s in a closed internal In.] hl1 ~ (2J tt.y:) is if if/ert?- n t i a bf,~! in. t he o/Je 11 in t erual ((J.. b) .

1'1u.."iH there exists at lco st one JUlin! c in the open intcrra! (fl, hI ,S,U(",h tha!

r (" 1 - r ~ (J ~ I'

- :::. (c)

b - a ..

Pr:"o('Jf,: Let us consider t ho tu .. net.inn ~r l-'f) d.e'~:~n0d. by

') , , • t i.

It i,s uxsu nu .... tl ~hat Ity·} is cont.iuuous in lu, hi anfii di~Jl~'rpni~riahll! in (a ~ "') ~ f\] s u, \ v ~" kn u \ v 1. h ;1 t k· xi ~ eu n ti n uu u s ~ in d d i fre rip n ~ i a b II e

fur all _\" ~ 'I'heretore .. it iollows that ([1 t,,-), which i.~ the d ifft"rc'nc{' of It X) and h:"t ~ i S L n n tin u U II sin [0, bra.n (I d 1 rrl'1"u n f i a b 1 -~ i n u ~ b"

f rom (i) \'V ( fill d 1JvlJ. t

n t (I) = r ( tJ) - J~ 1I ~ a 11 (I

01"

J~ ~ ( ~ b) - l (u )

.~ .

i> - (I

___ (jil

1-[(_~nl"L~i if we choose t.h« cnnst a ut l: :JL~ gJv~n by {]~}~ then (r (Xl satisfies i;lU th~i conditions 0' the "u~lll"t~K th~,~urenl in l(l~ bl ~ There .. Iure (by Rolle's the urcm l, L hur« cxists nt I -'ast 011l- point (' in t.~ ~ open in"rprva] (0, (,) such h' t (~' (e) - n ,

Fro m (1) an d (i i ~, »: p fi n d t h u 1. the r (~S u 1 L (p 1 ((')

to. I" J r ( h) - I ~ 0 )

(C I ~ I~ ~ - -

. - k - b - (t ~

, . ' (ii 1 ~



",Iiv)

(roJ" ('~ n,,-I ((',

... .Iv l

Remark: .The ]Ja~r~ •. ng.l",~ Mean Vul ui Theorem is 'I direct generallz3 .iou 0'. 'I he' HOU'e! 's theorem. As sun'] . lJulh nl' t hf:' tw , c nth Lions incl II dpd i 11 t he s Lu t 'I1H~n t. q I' th f~ th celi' • n I arr: Bt:t: :]'SSi"U)' for t h t,_J Lh 10 rem ln hold. H '" ~ u add i Uun l h lhe.~t~ condi lions. "NC have r (n J - r r b) a~ "~e 1 L then we gul hack the Rfd.h/~ Thenrom.

- h l I 1 p ngt h of t h ( , j n te rv a 1 J a~ b~ i;; h 1 'Il - rr 0]'" I.,:: n + It. At ~ 0, i r \v p ~ 8 t U =-

b - (_J "'

.Alit t',~ If

o ~ 0 < 1 and c = 0 + ( b . u) .. =- r 1 Hh.

h=n+h

Itii~ 1 , rf i 1.6

Aecr rdin}.!J.v .. ~l r- L~llgl'·an}.!l·~:-.i Jrll(~an \;,I~Ul' rl~U'rln III nl~,\' hl' ~tat. ~d in f Ill' f(·,no\,\r'rI~ nltprnn' IV{_" Itnrul:

12'

Suppose a ]« uction. /{x) natisfies the .fullolving tuu) conditions: (1) fi,)f) i» continuou» m (I closed inierna] fa ~ a -j!.. h] .

(2) r(x, is d irfo~rr.'n lie" ble in til e ope n in teroal (u i nil) '.

Then there exists ai least OHe 0; Lilith 0 < 0·< 1, such. th-at

Geofnet,~ical JlIle,a"i,lg

Like the Rolle's theorem, the Lagrange's Mean Value Theorem also h,3B a simple geomcericalmeaning.

I

(fa) :

II I

~~--~ ~L_~ -+ ~ __ ~.~~

o u c b

~ . I

I

I

11b):

. II

W ~

F£gu re i. 7

Consider the curvp lei representing the function ~v == li.:1t) , with A and B ns end points. Then if x = (1 and .,. = {, are the :t -coordinates nf A and B ~ then

, fib j _. /_ .• (al

~lopp of AB == ,~ I .'

I) - a

On the other hand ~ if f) is a point nn the CUf"\I'C with c as the! x-coordinato, then I· t (r ) glVPS the the r;loJ1P of tho t.angt-nt- to the curve at iJ.

f( II ) - r ( (1, ) - t· J ( ..•.

I -. c)

l - CJ.

b5, that l11{~ tangl!ol tn the curve C at P is paraUp] to tho chord /\8 .

.1" Differe1'1rtial Ca Ieul.us:

13

Thl18 ~ in gennletl'l cal terrn S ~ the I JagruIlge ~s Mean 'l aluc Theorem states as follows:

If a curve ,C ill the xy-plane with A and B as end points is Sl ell that (i) the curve is continuous everywhere including the enrl points. and (ii) th · curve is smooth everywhere between the end points, then t.ht:·[~e eX]SL!-i atleast one point POll the- curve (between A and B) at which the tangent is parallel to the chord AB i

Remar'k: There may exist more than one point OJ] the curve at which the tangent is parallel to the chord joining the end puirrts; sec Figure 1.8~

y.

o

b

.Fip:ure 1.8

Example 1. Ve.ri[y the IJugrange"s Mean Value Theorem for the follvlving functions:

(ii) f{x):= e -x ill r - 1, 1]

(iii) ({x) = Sill - ] x in [O, 1J

» (i) Here IVr) == (.r _. l){x - 2)(x ~ 3) . Thr-n

f ' (x) = (x ~ 2) (x - :1) + (,:r - ],) (x - a) + (x ~ 1 H.1.· - .2)

= 3x:2 - 12. r + 11 !

which exi SlS for all L\~. Thl]SJ f(x) is different i able and thurcfor« continuous for' all .c. In particular, /(Lr) is cont.i nuous iu to, 41 and differentiable in (O~ 4)~ that .is, {(X) satisfies lJoth the conditions n[ the 1 ,jnf .. rrangr-'s Mean \r a] ue Thpnl'[:_1111 in the i nterva I l«, b 1 = r 0, 41.

L' (b) - _[_ (0)

t'.«(,) ~ " ",{i) (') ,_ a

1 ~t' I

]'~ _ r (4l.=- r (O_) _ (j

4', n

.

I. ("'Ii

• J . .., ._

, )('

0,

:l+ Y1.1~- ~6 ( 1)

- i} 1 I -.

-. ... - -I··j<

b ..... ,.)

\ '

2 (14- l/\,r:f), tho rnsult (i) hn ld~.

'l'hus 1 the La gTClllges Mean \7 al U II then rem LH ve ri I icd for It x):::; ( x - 1)( x ~ 2) ~ J.- - ;3) i 11 I 0, 41 .

("ii) Icrc, l'-,: ,.;; ~~' -_T. \V. firrd £,ha (", J_-) - - e ~ exists for ull .x. As such, It.-c) is di ~rcJ'e]l,ti able and therefore contin uous for

~I I 'T l I" ) x t . fi- btl. tl d m t . f tl

a ~ _ .r . nus, (JC = e Ha -~.S IBS I ot 11 It(:1 con, ]-10n8 0' le

Lagl"'an~ -"s ~ f( ~ n Value heorern in he in erval ~(1 ~ b J = 1 - 1) 11.

N ow, 1 ha rCSlJ lt ( i) holds if

(tl) - ll- ~ ) - I ] 2
- I e tll 1 ,r •
e - -
-
~ (- ~ , i) ~) -
.... """,'Ii" • ~'

{' -

or

~}c

J ...J.

t ' - np;' Ij

e -, ~ m

0.1 n 1

,.~~ii]1

Thus ~ the Lngrang~ 's Monn Val uc Theorem is ve rifiud for IL,-} = r - .r ~ n ~ -] ] L

1

(iii l Herr- I?x'~ = sin l,,\". 'l'~lu find that. j't{x) == r- ') exists

\111 - x-

f(~r :.lU.l." ~ l.·~ lence l .r) is drnrrf'reml ~ab~~ .. in (0. 1). j\~s,o" ~.Xl.i-' cont.inunus lur all .r . Therefore. l~o1-11 th ~ coudit.ions of the La~-!l';'lnglj' ~ Menu \r u lu« T'luurcm are Rilt.i~ lit ~d ill the interval 10, b I = l U. 11 'I

'I

(J_~ - r n~) ~

J1

I I I

\ I iT .

, iI

,-

f(

Of these two possible values of r::1 only {\/rr.~ - 4 )/n: z: 0.771:2 I ie~ between 0 and 1. Hence, for C O.771~ the- result ~ i l1101ds with 0< c< 1.

Thus, til Lagrange's Mean Value Theorem is verified fur t(X) =- R in - ] .r in ! 0 , II.

Remurk: In ench of UU:." C~l~(~H (i )-(iii), we can writ.e down Lhe corresponding value of 0 by using th{l Iorrnula ( - (,' - tt Thus, in ease b - t1 .

(i), for example,

i' 0 o -

1- 0

Example 2. II:" r ~ t1C} > 0 Q I. ul! points of" a n in. ie rual prooe that /\x) is niricttv i ncreasing in the i nterual.

» Consirlur any two points x 1 and x1 in the interval lo t h! with x1 < x2. Employing the First Mean Value Theorern to f(x)

in lXl I, xz' , YIlt: get

[u, bL

Since {'Vi) > 0 in la ~ b 1 we have ( f (c) > O. T herefore, (fx2) - i"(x1) > 0, or r (x'l.) > ,. lXI,I. Since x2 > x I are arbitrnry.

it fo110"'5 that f{.x;·) i~ all increasing function.



Exampl(~ 3. It _"t > U, prooe the [olloioing:

( a ).... .,. 1 (J -I \

1 - < og" . - J.:- I < ~1'

+x

(11) 0 < ]-

log (I -1- _r)

1 < 1 .r

» Employing the Firxt Me-an Value Theo]"P111 1u thp function lex) == Jog' (1 1- xl over t.he irrtervn l I 0, ~~J. 'Vv'C get

0< 0·< 1

*

r, r.

\7 log' (1 -I :r l == In g 1 + 1 ~

Hr

x

-_

1 -i Hr

. . . (i.,

Now, sinc« x >- 0 arul 0 -<: H < 1 , wj·-' huvr- .x> Dr> {) ~o thnt 1 -I X > (1 I Hx) > l . ( .~ r

16

7: U. Engineering ~Wa tIU'!IRalit:.~~.ll

X .r

<---<x_

1 I.X 1 + ex

Using (i) ~ th is i 11Hq U a li ty hecoruea the req uired res ul t (a).

Next, from (i) \V~ get

"I 1 + Hr

1

- o.

log {I + x ~

x

Since 0 < e < 1 the required result (b) is now immediate, I

J Example 41 If a < l), prnue tlia:

b b-a

- a < (tan - J b - tan - I (I) < 2 '

1 + b2 1 + (L

Deduce that

(i) 51I + 1 < (tan 1 ~) <" IT -I- 2 .

20 . j

(ii) ~ + ~~ < tall - 1 ( ! ) < : + ~.

» Let fl-l:_) == tan - 1 ~T. Then f' '(x) = 1 'J ~ Employing the 1 + x'"

First Mean Value Theorem to Itx) over the interval [0" bL we get

. -lb -1

tall .... - tan (l.

b- a

1

(" j • . • I)

~ 1 + c

Sirice c > u" we have ] <: 1 I and since c < b ... we have

, 1 I c2 . 1 + a·2

1 1

--~ >- 'J . TllllS~

t + ('~ 1 + b"

1 1 1
< < -
b2 ~ 2 ..
1+ ] + t: 1 + a III view 0 f (i ) ~ I hi s becomes

1

tan - 1 b _ tan - 1 Q b ~ a

1

<--- 2 .

1 + a

Since b - (1"> O~ it follows that

,17

b - Q_ 1 1 h ~l

-'I') -< (tan {-, - tan (J ~ <-

;t;. I) ~

1;- b 1 ~- n"

u •• (ii)

as required.

Fur (}.....: 1 and b = 21 l~lC inequality (ii ~I becomes

1

- 1 ]I ]

(tan :2 ..... ) < _-

-1 2

< 5

or

n I . _ - 1 r ~ 1

+ ~ < 'ta u :l) < -I-

4 G 4 2

or

fiJI; + 4 - I 2 IT, I- ~

20 <:: 'll a n - ':_} < ~ 4- ·

i)

rt ,.)

_+y<bu

4 ~,5,

Exercise's.

1.., Verify the Lagrange's l\,Ie{lll Value Theor 1']'],1, for the fol1uv~d,ng· luuotions. Find c in each case.

(i 1, ((.tc) == x(:t ~ IH,-~ - 2) in 10" 1/'21

(ii) {(x) =: ;1:2 - 3,x - 1 in l - 11/7~ 1.3/7] (iii 1 lt~· J = log ,,\7 in [l~ (J I

(iv) (t:r) = CO~'2 x in [0, 1[/2]

) fi· - 1 ~ ro l'

tvtx);;; tun .1:111 * ~

2~ If f1lx) « 0 at all points of an interval la .. b]) prove thal (t.t) :W$ strictly decrees ing in the in terval,

,3~ If f(:'c) is a Iquadratic function, prnve that the value LJf e in tho Lagrange's lVfe'3u Value Th eorem is equal to 1/2~br an ~nlcr~n:ds [0 ~ b l.

5~ If a < a < b, prove' that 1 - ((lib) <: log (bin) < (bin} - 1 .

D\l~[hH;(~ tha l ( Iffi) < log' t J ~2) « (1/,5 J .

Answers

(j.v~ r: = (1 /,2~ sin - 1 ~~/([ )

.18

o CD)

.r \li 'l!)'\j ,,-lP \H'n\Jl~ ~ ~ ;\_\l·fi.\li~.d ~ \, ,·,:;t \l~ 't\~ J.>X~'t\\"\,

known a8 th{· Cauchv's Ml an Value Theorem n::ullP,d after the breot F·r·PI "1 r at e-mu I '1;1[1 A~L_ Cauchy l17H9=1 :'7) ..

~ I, (>0 rein ~t I~j n I ){JO ..... ·.l J n ~I) 11J lit' t inn ~ I~ J() au (/ (; t' (x) sa li ,~f:P IIH I

loll U I tu n g /. h n -t' con ( fit 1 () n 1·:: :

12' frx and ,11 .\-1 (J rc {/ ifl')t· vitial. If. in III open int "Jrr al (a, b), I~i) R' (x) *- o in, t'a~ b)_

Then {herr. exists (It least one point c in the open mtcroal ia ~ b'}1 sut·I',' that

r {b} - f (a) [' (c)

- - --

l{(b)-g(a) .q'()-

Prf~ If:, J let U~ C nsjrier t! Ir' function (p r l d J- It_m, ~ hy

(~ (.r) :::::; /(.:r, - k a {x_~

. . . l i)

wher c h is a can slant to b(~ l~h(H.,{:j'n appropriately.

F'r nu the first t"~U of t~u.~ urivl('ft cond ~ i ins Ii f()] HWS _' hat qi l:t' I ;i:;.; C011 ti n ID.1 n u ~ i 1nl lo ~ b J an d d i fl~ rren i a bl . in r a. ~ I)} ~

i\J~O~ we find tl at

(P' l u) = f l CI - If M (a), tp (b) - /. ( b - h g (b)

Tln-refore. tp(a) ~ £p(IJ} hnlds if

l (a) - I~ g (a) = l (h) - I?g'!)) ,

If = j'f/!l= r oJ

, ~g (In, - g l a )

. , . (i ru )

He-nce, if we c] tJO~t· the constant I~ :l~ sriven b.v ii ), then lp (.r, ~~- tisfics all thp thrue conriit.iuns ofthe Rolh 's theorem Therefore (by - ,UnV'!-l thoorr liJ~ lhrPl"' ~ pxi:-)ts iT .~ ItH:-\t tHllf! point (.' ~n th I, 10f e-n interval ,a , b) such that (f' f((,],: O.

From (i) t: nd ~ ii ~~ \V(~ tinrl that the rr-su lt (11 ~ (c) 0 j~ tho snm«

or

19

I' ~.b) -_ r (0., ~ (b l - g (fI)1

Ii ••

. ~ ,. l urn



Remal'"liS

(1) IJ. Lhr- l"lgbl h.rnr ~Jd I of l.lu1 l"t!suh i iii i, \;\lP liuvr .. ~ (n) =t.J:: ib), B,ce~HIS~\ if g ~a) - .~ (h) ~ 1 hr-n .~ {xl s~ ti..,n~~~ ,aU ~ ~H_· ("HmHI~t mons or nil" l{nUe\~ theorem ~1Hd we ~.~·t!.L ,!;T I (c') - U ~u· ~tunE' c hi (rr , b} \vhic:h is nul l.rue. III 11:1S ht·f?~'n a8~'Hull~~d that ,~~ ~ i,X) 1:- U for uny x iu ta, h)1

(2.1 ·rht.1~ 'HId." lXl:; nun' I' lha~ nru· ,. in (0 l) ~~}r which l.ln result {;~wl

holds.

r {b, - r (I r I' ~ e) =:

I •• dv)

h - ([

Miter. If th I lpngth of t11(' interval fa ~ hi is h., we have II == b - i1 su. hat b - fI + It. Alsn. il wr se ~ H == lc - IfJ )/t h - u)~ we have [) < H < 1 and (~== a -J- Hh TAccol"diinul).r, the Cauchy's Mean Value 'I'h iorern may bp stated in tht~ following alternative torm:

II' tum Ill. H ction s fr.x, (1' ~ (/ .. ~ l[:r) a re (~t HI ti n uous in r a ~ a + h I an. {I tlitt'eren~i(lblf! iu to ~ (I + Ii} u-ith II r (x):;t. 0 in to a + " ), then there Ixi,r;ts (Ii least one '0 fnilh 0 < ~ l,..r 1,i..iu·1l tluit

t" ( u -+ h} - t" ( u. ) g ~ II + II) - J! (0

l' J (0 + ~h) g I (,(1 ...L Uh)

.•. t v ~

Exampl 1", '\ prill' the ("ou('lI.v's Mean \ al uc Theoret» for the [oiloto] n,~ [unrtious:

'1 I)'

(i) {(:r: ~ = J.~. . ,Jl (_:r-) ~ x.... in i] ~ 21

( j i i ) Il,y) == ~ i ux ~

r r ~ TI'};;;;;;: "()~ l'

_.._ ~ • "ir= L 1.. ~'~

L I _ ,

,.) 1l:/.~ "

: ~'} 'J

» (i) T [prf:, !(x) x, J< (.r, _lL- ~ sn thul {'iX' ':: ~~".:- nnrl

I!.: ~ (_r);:;: 2x~ Thus. ILt' f .md } ... { l:r) nrv dillcrcnt.inl ~ILl and th(lJ'l~fnr(J cnntinu JUS for nU x _ \~SO,..~; (.r)~· I in (1 .... JI) • Tlu-roforc :~H ~ he

20

conditions of 'the I aUrcJ1Y · Mean \, ~ II re TIl, orern atl satisfi d in the i nterval [a'T b 1 == 11, 2J I

_ 0\\1. the result

/~"{C ) g '(c)

{{h, - {(O)I g(b) - g ~.CJ.l

2c

_ l (2.) - [(1) ~ 8 - 1 __ 7

-

g(2}- gel) 4- 1

14

:3

-

l ,~

of I +

Hence for c = 14/9 the result t i) holds, and c lies iu the interval 1, ) ~

Thus, the Cauchv's Mean Value 'Theorem is verified for

, ""

((x) - Jl[:~ and g I(X} := x2j_Jl] III '2J.

(ii) Here ttx) ~ e" and g (x) =: e - x , which yield t~'(x) == e~'( and g f. (J() = - e - ~. T!llUS, f{x} and g (.Y) a.I" - d ifferentia blv' and therefore eontinuous, and g 'Or) *- 0 ror any x ~ H nc ((x) and g (x) satisfy the conditions of the Cauchy's Mean Value Theorem

in ev rry mterval La, b] .

N OVl, the result (i) holds if

nb oU

I,"" - I~

_ a

e-h_ e-a

-c

-e

.

2t. -e

OT'

1

C' == -_ 1(1 bl .J 'II.

TllUS., for I(x) == eal' and l~ (~r) == - ~-r defined in any interval

tn, -J, th I' auch~~'· leal} \la ue 11'111 rem j~'" verif d.

(iii) Hero, f(x) = sin .r and

r r t\~).:= cns x and if j ~Jt) ~ -~ln J:.

;::. (x) = cos ,r \v hich yi ld

J-,..'v Ident lv, the Cf) ndi ti 0118 of

..

the Cauchy's I\1l~an Vi] lue Theorem aru satisfied in every inte-rval lo ~ b]~. ~v ith 0 < a, < b < n:/'2 .

Now')! the resu ~ t (i) holds if

.

- S~Jl ('

cos b - cos [I.

.

t.e.

- cut c -

.... ' (b-u 'J (b+U)

~ srn - C!os",

:2 ~'

\ ;

. (b - tr, J " (b +~J -

- .) Sin SUl

.... ,,_ - - 2

,.,;;, _..;.

( b +" a J

+cnt -

_ ~

or

r: =

1

~ (0. -l- b)

....

... (iv)

1 -

Hence, for c == _" (0 + oJ

. ~J. _ ,

.o.!

TI1U,cl, the Cauchy's Mean Value Theorem [S verified for the given functions over the giv,e:n interval.

Note·: In t:ases (liJ ,and 'UlliJ'~ tlu~ V~II~ue of c is the arithtnetie mean uf ~h(:" values of a and b. •

Examp l e 2 I, t_' .

..IJIi..:" ,. , _ . " Ii

I ~

\)1 1 ~ x 1. ug (1 + .r)

............... -- < < 1 ~

V]+·X ~,-l

SIn ~lr

» Consider the functi OIlS t(x) == log (1 ~I .r ) and g (x) = sin - ] ~( , B,ot.ll of these are contirruous and, diffurcntiable for 0 < J( < 1 . Thel'(!fO'rp.~ these satisfy th.1:? conditions of the Cauchy's IVlpan Value Theorem in the interval [O~ Jr.l ~ "vhere Jf < I ~ II~[lCe there exists t.J. r\fdal11U m bp.f' t: betWQCMl 0 and x such that

{(eX) - t( 0 J~ _ ,{ ~ ( c)

glX) - g(O)1 g '(c)

log i 1 I :r) - log' 1

1

sm

1

X

- - 1.

SUl 10

:1 ~ ('

.or

1,0 a (l -I- r .~ "\:' 1 -- r

1

AUl :r

• I • t i )

22

T. fro Elrgiuf;!(I"'ing' JWnfllel'l,utics-ll

, I ,
~Il -{' 1 and \11 -(' v!l ~r
---;:==: = < I > I
v l + r: \' 1 , C \1 ] +x
I r
\11 r \111 - t:
llencc - ~ . 1 ( ii )
~ <
I .. I , . .
,11 ·1 . ~ \111 +. ( .
» -;

~,Il +. x

«, ~ ng (1. + _( ) < 1 . -]

sin x



I" \/ er d~r the f\u Id:b.vr~ ·I\'1t~an \1 n h.UJ T heurvm for the fnllowing fun l'1 inns. 111 (!Ht'11 f'f I S( i ~ find una IJ proprJ all' f' •

I -i i ~ /1:':::) 1 . ~ I/.r . 1 1 I') 11-
- 'X'''' ff{.X ~ - III
- ~ ..:....I,
, r-
... ~ (l.t) . [ 1/4, 1] ,
(LU I,J'X g~,y ) - l/\',r I.H
• - t·v) I'r,,') - sj n r ;1"1 I,(J - !l'os X

• , •. i ., h \. .,

in I

0] .

'2.. IT {} < (I -: II < TI/~, provo t.hat ( i 1 H <, (s in I) s i 11 (I ~ -: h - n

ti i.~ () <: l cu s a - (' u,~ b, -c b - a.

:-I. If 11x J is coriti nuous In an interval [n , bland differeutiable m the interval ll1, bv. prove that, there vxists ~ I i' wil.h rt < c < b such

th ~ It ~.f'! rt rt) - It h)1 = r ~I (') ( a ~ - h:!. L

A"l:.Slllel7'"S'

L. ~il t' - lv'fjJ2

tii i ' (' - ]/:2

(v) (' - - IT 4

1.6 Taylor's Theorem

\"filll now Fn'oCPt,:d LO llTOVl~ a g-rnera 1 !\/T pall Vuluc 'Theorem. or the u tf, Mi-an Va ItH-~ Ttuurom. known :::i~ the Tavl or'« 7"'11 (JdJ"t3tU ~ nuun.d aftr'r t.lu: E'ngli~11 BHl'tlH,~1l1U(·lCi;111 [-3~ro()k. ·faylol' (1(JH;)-17~11 )1.

1. IJifferenl ia 1 Ca lculue

T'h.eorern: ~'::;'IJ)lJ08e a function IL1~) suiisfie« lh» j;,lloH'lll!!.

C()},I( I ition s:

(]) {(.r) UlH! tt« 111',;;;,[ n -1 deriuulioes are continuou« in 0 cloued inieronl 10., bl "

1 2 r l JI - 1) ,~ I 'I' '. I I • I " I I

~~.) , (x) u\ (.I. [erent ia JH? III t.Iu: open inlcruat (a~,) .

Then there ~ xisis at least oue point (' in. the niJCII interval (rr , III uuch that

~

l(h)== /(0)+ ib> a){'(O)+ (h~~t) fnlal~ ...

.;..I •

. 1. . I~ 1 ( 1 II

(0 - a ) f~' ~ 11 - l I , ..0 - a ) t' (fJ J

---+ - (a) 1- ,- lC)

(n~])~ III

Proofs Let lIS consider the function lP (xl defined b~v

2

f" I /' I /' t (h - x l t" 11 '

tp (.t' == . C » -: { .r ) - () - .r ) - (x) - ~ r' [ , x) ~ " . -

~ I ,II - 1 tb _ .~,~.}i.I

,}-.X) f"rn--]~()"1 ~ l. (2

- - , x - if " ~ . ' ,i . )

in - 1) ~ '11 r

I·t " I l t t /" , t' ; , t (II

l~ assurncc .nar . (x)~, (J()., . -.

1) C'l;) are all con till UOUS

iu ro;~ b J" Alsn, tb - X )1" i ~ coutin unus for all 11 in every iriterval.

"

Therefore, from (2), it follows t.hat. (p (x) i~ continuous ill fa, bJ .

Further, since lid (x) exists in tho open interval (" a, b) for I? = 1. 2~ '"' n by assumption, and (b .- ~l._~lL' is diffort-rrtiable fhr /( == 1. 2" ' " , It in ~vfLry in l.arva l, it follo\vs from (2} that {p '(,x) exis ts in ((1."1 In L

Next .. we rind ti~onl(2) that l~ ({1) =: 0" Let U:3 choosP/I' such that '-P (a r= 0; i, e" ~

I)

( 1 t t' , " f tb Ii ), ~ J - r ,r i £,1 J'

L l -= <.p r a) = I {') J - : (a) ( J - a ) f r t: ) - ~ , ' ~,

, if

11 1 (I i - l: (} (I

fI !

24

~ h - rJ);~ - 1 r ~ n - 1 ~ ( ]

-"------ " U)

(n-])~ ,

! •• (3)

'I'hen {r (X) ~ati:rfi{-"H :111 thp conditions of t.he Rolle's theorem in I u ; b I. Therefore l there exis ts at Ie ast one c ill the in terv al (a., b) such that

~ .. (4)

NOW'f from (2), we get

b ,n - 1

_ ( - x) J _ t' (n)( ,:-~ , ,L l .

" lx)+r~r,.

{n- 1) .

Hence (4) becomes

other terms cancel in pairs.

bf1 - 1

( - c) 1 - l {n I( c) + h f -

(n - I) !

because b - c *- a ; in fact c < b.

IJ sing (3) in (5,)~ we get

f (In (c) = n ~ _, [n b) - f( a 1 - sb - a ) (' ((J ) _ . , , _ I b - a) I! . 1 r n - II {In]

(b - (1)'J (n - 1) ~

o OT'

,

h , "., (5)

Rewriting this, 'Are obtain. the required result (1)., This completes the proof of the 'theorem.

Remark, For n= I! the T'aylur's Theorem reduces to the Lagrange's

Mean Val LIe Theorem, For n -:; 2).3 J ' , • j1 Vie' obtain what are cal] cd 'the

o rr:« ,d':' T:" , 'd ".of" o . IJ': '1",:, T"·' '!:l "', ' ,

oecon :-, .1.11 ••.. WI eo n 1- a .ue " It or ems.

Al iter. Taking b - u == fl. and c === a I ijh, where 9 == (c- Q,)I(b - Q.), the r8Hl11t (1) may be put in the following altorn at.ive fnrm:

-l

I '" i' h I' ,,~

n . (a) + 2 ! (0 1+

. ,

I n ~ 1

.i: I~ 'n . . -I

(ll - ] ) .

. .. (6l

... o If-l': i I 'h~ ~ last. l.(~.l'ln uu l. ~~_. ~'ig;ltt-jll;.HH"J SI d(~ of Lh J rvs d t ( i) is k un\\ n as the n_-ulofllder alter n tCI'Ill.Y in the 'I'aylnr's theorem. '"rhis remaind r is en Bed t.hc r~ LUI ~ I in del~ in Lagr« n,Jgf! 's /fU"UL 'I'he rern u.i II d er lOU n be -'I"',\,'VTi~1{ J1 in I l_ .r ~~ ,:rl'ns ~i~~"'1 ~ hut \\re wil] no cu sider hem,

~

f{x.) = {(([) -Ix - a) (' (a) + l ~ ; ~Jl~ r: «(1) +

~ .. (7)

This ex 1I"t!'~SfOn g"iv s the exp insion ot tLl")i ir pu 7crs of (x - a) and the eXpSJ18ioIl contai ns n t 1 terul::L Let us d 'l1otp he sum 0("1' h firat n . I rrms bv F3 f.x I and th ~ last term by R Lx)"

n I . n

. J, ~ 1

~ f ( . ) {.I ( .\ ~o I {(II - 1) (Ii',)

~ Llf) =-' n) + f' ~ - a. I (I)". ~ ~ + 'I.i

1/ (n - I) '!

11. - 1

r

Vr - 01

r~

- , . lS>

:= ,-(a) + 2

I = 1

nd

R

n

( ~ )11

" - u /,,,, ( ,~ ) ['

.x)::::. ., . ~'[l+ U _l-U )

fl.

. .. 9)

Ii (.\" ,

n

. •. il U

N UW, sup [Jose t.h at It x) possesses derivati vel') of all orders an d that R (x) t<-H1ds to zero as n ~ (XJ ~ Thou taki ncr the limit a.8

" . ,~ .

11 oo un both side~ nf (10), we get (beal~ill.g ill mind tha the left

hand side lK independent of 11)

It x ) == lirn S' (x)

n

n -]

(x - rif_ f' (r) ( .... ,)

I . a ill

l"!

u~iug .R~

== f (a) + lim

r;:;: I

r' . (11)

The right-hand side of' (II') is an infi niie series' in. ascen ding powers of ~: - a. This series is called the Tovlor's series for the function f(x) about the point (1 ~ It i~ also referred to as the Ta~vlor"s expansion. of flx) in. power series about x == a .

Note: Changing .r tl) x ~ 't, and Q to x in (1 ])1 we obtain

[iii"]

{(l; + h) = {(xl + L. "~ /(tI) [xl n.

n -= 1.

... (12)

T1J.e right-hand side of t 1~) is the Taylor's expansion of f(.'· + h} in ascending powers of h) about the point .:1;" •

Example 1.. Verity the Taylor's theorem uiith n. == 2 for the [unct ion. f{x) == x(x - 1)( .. \: - 2) in the interual [0, 1/2 J ~

» When n = 2, the Taylor's theorem raads

2

f{h) = flu) + (i> - 01'(, ((1)+ (h ~~) f" (c), a < (' < b .,. CiJ

Here ftx) i~ a function that sufisfius the conditiuns: /tx} and l'(x) arc continuous in fa, b 1 ~ and l n (x) exists in (n:o bi .

We note t.h at Lho g i ven function tlX} = x(~'t - 1 )(x - 2) s atisfi C:-S the required con d i ti OIlS for all x . Therefore the resul t (1) is valid for this l(x} in let, b] == [.0; 1/2] . Accordingly, we have

27

. .~

'_ ,

We note that rCa) = 0, ((1/2) ~ 3/8, f'(x) ~ 3x2 - 6% + 2, SIOl that f' (O) = 2 and ('I' (x) =:: 6x ,_ 1) ,., Accordingly the

3 .3-

result (ii) becomes 8 --= 1 '4 (c - 1) which isatisfied by

C-I ,=:;: 1/": 6 .. ··

'_ _', ii

Thus, there is a c namely C == 1/'6 betwe:·'-:I 10 and '1/'2 that satisfies the required condition (ii). , _his ver ~ fics tl e result (i) for the given (r(.x) over the given interval, :.,

E,x,ampl.e 2. Prove, using the Taylor's theorem, that

2 8

) X. x 01< ..... , 18-·< .. ·· ... · l'

;.. - x + ,-~:> O,~

.'. ~ c. - 2 . 3(1 + 9)3 ' '.' ..... .... '.'

1 (1'

. a' I

[Olb ". [ [

2 3

log (1 +. x) < X - £2 •••. + --3~'

.... , ) .!

X- >: .. ,0"',

," .... ~

~

» For n ~ 3:- the Taylor's them-emvielda lsee (fil)]

.. :2 3

fCa +,~) == f(a) + X ('fa) + ~ 'I fll(a + ;'~ flU (, ~ 9h),

~ - .

= 1- 10:--' 'g ... " x'_ .

.._ ,_._. ,I

/'(.i':"'11 1.

: ,.'" l~ ~

'''/''.': ~ -J

, -, I

9 ,3 ~,

-~*.-.

Then- expression (i) becomes

1 I 1 ,.e.2 .'. 1 '"

. og (a + .. t)·:= I Ol'~ a + ,:- . __ ,+ 2' 'i I - 2'

la -'. i, a

3

,x ,2 0 9 '.1-

3 II 3' _- I'.. < ['.. -c

I ~, (0 + Bx) I

• ~ i., (ii)

. ..~

Next, \VC J10h~ that, srncc x > a artd a > 0, (l + t:tt I';;' 1] or

1

,,! < 1. i( 'onseque ~ ntly, I~ H) yiel d s

t 1 O:r(~

+

2 ~1



,» If (I = O~ the Tayl ors theorum reads r sec (7)1

II -1

i • (i)

Itx)

1(0) fo> L

r = 1

" ()

II') (I. ,. 1'1r

I·~· ',.l')' - - ';0.: an ,l.' \ sm 11· I- ~

\ " ' ~' - • ~, _ I • 'It - ~ •

dx ~

.

~lU J.

n - 1 r (

" _r . rrr

'" -- F:l111 ~

~ r ~ "J!'

. • .:..I

r = I

, I • 'i i l

i),
- I , .. :~ J
" L .l' . l rn t' _ r :1rr
SIIl x I~ln ~ + 9Ul U1 J.: I
t' , 2 ; ~ i 0)
I ...
I - 1 ~ ...
.~
x
- x ('n~, (Ort) o < 8, <.. 1
'l , \
! . • . ~ t iii l

and

.

BIn x ~ .1:

:1 .r

i) r

t 1 I

:1 ~

.'I: 1 '"

X - ~ I f ~ R: ~ H ~'I >- ;;--

,) I ",. "-, ~ .... ! .'\ ." j

~l • ~ .\ ~

, .

1.- -

I I X I _ .l~-

1 :i ~

r: .1

:r

5!

In vei \AJf of (iii lan d (iv), th ~'~c iueq ualities yield

:l 3 5

J; < .: ,,<., X .:!

x - () , - ~ I n ,:t ...::; J - 3' I + h' '

.. "]0 , • ,) •



Example 4.. Prnue tliai

- 1 tan

- 1 ( It" ). ~ • - ITI'

== t~ 11 ~4 + ~

. 1 + 1{2/16

» I ~I 'r~~ we 1 ave ; n expand Itt), == an - 1 in PO\V .rs nf (x - rc/4) - The expansion iF5 br-l von by the Iormu lu r see( 11) \.\1 ith a = n:/4]

rex) = l ( ~ ) r l..v. - TC /4) r ( ~ ) + I x - 2 ~!~l j''' ( Jt J

. .

Now, we have

f.' t 1

I (J\' ) - '. a n -.f_

1

T~11i81~ ore, {( n/4-} = ~ n - l fn/~) 1

~1 '

[+ x~

- 2.1:

TJ1PI'uforp, t "ln/4)

L ~1 - 1 1 (nI4.)

CU.I x - I all

JJL~ - n~) 1 + rr~ /16

r)

it (x_ rr/ 4, ...

4 ~ 1 -I- 1[2 / 1 n ) ~



Exa'l[ll pic 5,", Obtain the poun:« series exponsion Hf' cos .\- obou: x == J{/;~, Hence ti)u.l on upproximot J value o/,eos t1 }'I _

» ~ el, /tX) = e~l~.r r \rVt a1f"ll! requi rpd to exp: nd 1(.\-) i n powers of

(_"( ~ n I :1 ') . "T his P x ~J ~1 n ~ i un is g-i ve n b V th l~ 1'0 rm u In

, I.

,'. . /,( II I T I'·' (:\- - 1!. '::3" /' (U J r }.~. '

ILl) - . l ,'.J.' ~ •

J n: .")

j ~ I 1 ~. )

, . . til

so

I

L '"

w~ not: '. th,a-c"

,i

f( ~ ) = cos; = ~ and f<n) {xl = :~n (cos x) = cos ( x + ~n ).

Th,e::refo,re:, tl) becomes

00

1 ( '" Jit/3r:_ ( - 1lt' )

DOS X' =: z: + ~. I)i"'" ,I.... : cos' ~ + n ... ' I

9 ~ " I 3 '2;1

..;..II ill" '.' ~.

n.:= I·

t : '.J ~ ., ,. (n .. :

( J (' ).2

1 f[ ,. K ," n' 1t

== ~~, x ~ - .. ' I SIn ~..... '-:-,' . ,x ..... ,_ (~OS ~

no ,~ 3' 2 <!l) 3'

~ u I,::_ (Ji '".1

. , .

( J3

1 it - ~ 1&.

T' 3. x - 3 sin :3" + ~ .'

.

2 :)

1 .. la (., 'it )... 1 (.. 'n J" '-J3' (.. 1t J:' '," "

= 2-2 x-a ~ 4.3:-3. +12IX-3. -I' ,--

~ ~ ~ (iii) This gLv,esl-' e Taylor's expansion of cos x about the point _.~ =. n/8~,

To determine it' 10 _ 1t

x~ ,~~.

. - 3~" ~'180

cos 61" ... ! -; - 180 - 1 (1:0 )2 - ·..J81- ~ (1:0 )3

~ ~ , .. , ,I ." 1 '.... .. .. ,

1. 43 {O' 0 ·17'·AI5} 1 (0' 0174" ;oo;,~2 fi' (0 0" I' 74- 5·)3

~ 2 ~ -;;;} :- .. 1. I ~~,' ~ ~4-' ~I .' :0') :, '12' ' .,' '!' .. 1,1_' " " .

i:;, , ...,. " I .~., I '

CO:'~' ,6' ;11 0

- : ,~I. - 'I

let us

take .x: ~ 610

- 10;' 4"; 8-:4" 8:,·

- .. ,::,!!!', ':'., .,.::, '.'

Ex 3 mpl e 61t, ,0 btain. Ta lor's exp an-sion; of' the function sin (x/4: + 11.) in ascending pourers of h upto the term containing

,h " ,. He .. nee deduce an. approximate ualue of sin SO°

... (i)

,31

"n 'J'l.

sin, (.x + ,h) = sin z + " ~,d '(sin z)

c: .n r ~., ~."~.=",,,n

,1. = ]. Uk

~ ,:n (. J~'

h n'",-n" '

~: sin I + L -' r; -J' sin x +. ";", I

n .' ,;OJ

n=1. ,

., ., ~ (ii)

Taking X' =: rt/4, and retaining terms up to ,h 4: in (iii), we get

~ . ~ (iii)

'This is. the Taylor's expansion of sin ( 1[;/'4 + h~ ). upto the term containing' h 4 ;;

T kj"' }' 5 ' ' Sn: x

, ,a: ng .. '.,' ,1. :==' ,:0:::::: L 8 .... :'l\'. ~ ,_-

. v ,36

A 0872"7 di ,', (i ii) t

,,',' ,',: t : " rue Ian. ill i.111:i we ge'

1 [ '1 1 l' 'J

" . ,.".:. 0 " ' " . '. F.I!',. ,',,' .' .a- "',1"":'" ,,2 . .', ":;' , " "( '. 3: ',' ,~., .' 4

sm 50 ~ "1/2[1 T (0.08 (27) - 2 (0.08/27) -6 (0.08727) + 24 (O,OS727) .

O 7'0" '6"'0" 46:'

r"5'!' :'1', . I"'

I-·!~!" .' I '.' •.• " ---: ';"' 1 _

+ . '.,.. ~

Ex', ere' z~S"e' 'S' ,

" .' "." '

:',' '."', "l', :.._. ','

L 'Verify the Taylor s theorem with n =. 2 for l(x) ,:= (1 _ X',15/2 in the in' erval [o.J' 1/.3 J .

2.. If 0 -< x <" 2 prove- that

? :'i

(x ,_, IJ~ ex,~ 1)"

1 og x ~ (x - .1) '=' '+ ....:..,___--=---

2 3

Hence sbow that log 1'.1 ~ 0.0953.,

32,

:~.. F:XP;,HIU si 11 x 111 powers or Lf 1I/2) u ptn the torm cunl.ami ng

·1

(X - ~/2) . Hence lind an appruxirual t-! value uf sin 01 n •

4. Ex P au d !. a 11 .r a J-'H j U ~ ! JU_I po i n t x ~ 1114 up Lo the t erm eo n 1. a i 11 illg

I~

~J" IT/·'l} .1-fpnCfl find an upprnximate value of tun 46°.

5~ Fmrl UH~ 'I'aylurs t"XptHl~inn of log (~O~ _r about the point J'~ = 1(/3 , 6. Pruvr: l hu l

( 1:-) H;1 10 ,j

(i J tan : I x J ~- l + 2x I ~x" 1- :3 x + :! l" .

I (l1 J 4 ~J 4 {1

(ii) ()g tan_ 4 -.I J: - 2.1:" + 3 x -+ 5 x +

7., ] )rovc the fo.l] O\V i ug:

h'2 h~

(i} ~in (.\' -I- h) = si n .r + h cos x - srn X - cos x + - I

t~. ~ :3, !

J:P

(i i ) log (X I It 1 - I u rT h +

t"I It

~ ~r

')

211 ....

3 .t'

2h"

I.J 3

r :2

+ - cot t' co:sec x

:3 -~.

( ,- 1 1 ,.

v) :O-:1I1 i Y -1 ) - s: n

or

IJ 3 (IJ'

~: 1 - x'_) ..oJ

2 It

- ,

?,'

~ -

. I . I •

(V) ~ tun LY + ,}) = tun

]

.. t

h

.

~ IH x::_ J;' _

.J' '/ I,
y .... r~ f~
I _r 1 't 1 .'
< cos ,r < 1 +
I) 'J "2 .. J
..... ..... \ .. -

c I I ::.::

I ~ l' - 3/2':;

'J

/')' (.".

T{ ...... \

_~ -

4 • t. I u .r _- 1 I- ~ , r - IT / :1) 1-:2 (y

ti~H ·'16 -O~~!J ~J.

~- ,~

5. Jug cos x - lug t ] l'l) - 'J:, .t - 11:1:~' ~(x Ttl ~~ t'

~ t ·11 ,I ~~ } t x - It / .:1 ):' .~

From Section 1.G let UH recall that the Tay lor s expansion nf I~ x ') abo ut a pn i 11 t ,:r == (t is gi,veII as loll ows:

Ax) -

/,(a) +

n {_,- - a)

--..

. . . (1 )

I~ - 1

As mentioned ~arli ur, th iH ~XI)~LllSi on is valid under tJ1P suppositinn that thr{_lo remainder .artl -r the nth term l(_'nt~~ tu z,e rn as

I I, f,XJ

If we take ,n. = 0 in ( 1) w ~ get

~.

ftx)

... (2)

This expression givus the Taylor's expunsion of /{.t") about the point:» -;!: O. T'his exprmsion 18 known as the .At1clcluurin"s [r<;\p1f.,rn."iioll of f(J:)I, n,(_U11~~d aftc J~ ~il(~ Sen l.ish flliTf hematician ~nlin Macluurin { 1(j~H-17 46 L 'I'he infinit Hcri(~~ on 1 he l'igl1 t-hnurl Kid~ of (2)~ \1.,11- ch is a pO\lve r ... ,cri, IS in ._ s ·pod i,us po ·rpl~S ~.f _t' i ',. , 'all ,~( ~ the Mac lcusri It seri e» r( ~ r ~ he fl1.11 cti (H 1 It x). Th LL I"PH til t ,2;) is retefrt.~,d. t ) as t.he fCl(~/a,Ul··i"1 ''II~~ theorem.

Let lU3 set tt ... ~) - y ~H tha 1 (, D) = y (0) an rI 1.1."-;(" thr- follo\vi ng nutations:

t" (.r) - " lit )j .y l' U
. ] .
,,10 ~" r -1:' , t f'{ n ~ {n}
I v ., "
.' oj . .
,_ ~ T.ll. ,Ellgi'n.ee,~illg Muth.elll(ltics-ll

r(B) (.~) f.··· (11) (O ) == (0")

. X ==YIJ~ ~ L'Yn .....

T11cn cxpruss ion (2) takes the form

Ij

Y (0) -1- ~ .:!_ .Vn. (0)

L.-i 111 n

II = 1

~ .~

x x"

- .v to) ~- XJJt (0)' + - - " . (. O} + v ,(0) +, .

. \ .. 2 ! ·!'·2 . 3 l ;1

Thi s form of the Maclaurin's expansion is convenient in C{ rmpirtatinn al wr irk.

Some st andar-d Maclaurin serfes

Below we obtain Maclaurin's expansinns of sorne standard functions:

(1) Exponerl,fi'al Se, .. ies

Let us tak.e y == eX ~ Thon :v (0) == eO == 1 . I\JSO"

In

( ')

. ~, 0" 1

Y - If = e ~ V (.) -= e = ' .

n: J n. ~ fil

::r

Hence, the Maclaurin's expansion of .v = eX is [using (3)J

(:..0"] L'C'

vJI .»

] _L L····· ':_\_' - L'" . .,;",..'

----n _ .,J -

.. ' r . ~

. ,n. .' Jl.

fj~l fI':i""O

2 ;'3

. ,x ,x

1+x+2~' 3l'+

, J II

. ~ . (4)

e

t-=:(:~rH)~ on

the right-hand side of (4) is known as tile

. { .

ex pone nita. se rtes.

(2, ) Logari Lh.m. ic ser-ies

IA~i V == log (1 + x" Then ~V {O) == log 1 == o. Also,

so th at v (0) - (- 1.)n (n - 1) ~ ,.

'n

expansi 011 of .1' =- log t 1 + x} is

I Ion C't~ i th e' [\.iacJ au!" in' H

log (1. + X)

n = j

;:: x-

4 Je

-, +

4

. - . (5)

The series on the right-hand side of (S) is known as the loga rithmi c series.

(,3,) Sine Series'

Let y = sin JC ~ The-n :v (0) ~ o. Also,

,( I" . ~. ( nIT) tl t 0 -, l" II Jl )

'II. = --;;- (sin xl - sin :1: + ;) "' so tnat Y11 C I =- sin 9. .

,~ dx ,- -

Therefore, the Maclaurin's expansion of'

. .

.v = :51,[1}C IS

D>~_ N· (' J )1'. C

~ ,~.it. n.1t x ;, x~

£·an x :=: L ~ I 81:J'1 ? == x - ~ + ~ I - .. ~

I ~ , ~ ~l " ,.).

n ;;;:] ,

.. , .. (6)

.

sertes.

< 4" Cfl~"in. e oer ie«

L 't J! = 1[: os J.~. Then )fl( 0 JI::: 1 " A 1 SO~

JI ;;;;:. cos (x + ~,~ J I so that }' (0):::: cos ( '~1l )-

I.,!,!& ~

Therefore" the Maclauriu's l--xI~,al)s:ion of .,f == DOS.X is

I:>..:I! ).;.n ( -" IT J 2. 'I

',~ .. ns. x 'Ii'

~ l~~+'~

n r 'co S , - 2' == 2 ! 4: ~

n ~ 1

".f)

J( 7)

6 !' + · , ' ... (

Ttle- ~e ries f) It the ri gh t- hund side of ( 7.l is k nnwn us the cosine

.

series.

(5) Hirurm.ia] Se,,"ies

[ 'nl iI"T"] \1

,;I(.~ L J' - (1 + .r) . 1,1· n, .\1 l 0) -= 1 . Alsn,

v == nt tin - ] ) 'U1 2) _ -. (m - n + ]) , 1 + X )'11 - n

o n -

Rot hat Y n ( 0 ) == n I im ~ 1) t n l - 2) , 0 0 , 111

n + IL

'.0

1 +-

" If! (!Il - 1 ) (In - 2) . - - (In - n --+ !2 x"

c: 11 ~

II - 1

711, (,n - 1):2 In (111 - 1) Un - 2) .J.

1 + IJ1X 1 2 1 Jf + - 3 ~ x +

The ser ios on the right-hand side of (8) is called the Binomial

~

settee.

Rema.rks

(.1) lf III is a positive integer, the f:criet-l on the right.-hand Hide of (81 can tai ns unl V ni I 1 terms. Beca use, in l.h is case, .V - 0 for n >- 17~ •

o· .~ 11

(2) If In is no l. a positive j n ieg'cT \ then the series un the righ t-hand aid e is an infini te series,

Example I. Expand the l()11()lO~i11.g {unctions in ascenditu; pouiers of· x by us ing the Macla u rin 's tl: eore m:

(i) tan x

(-")' - 1

11.' tan (1

x)

(ifi)

x 1 + e

..... a\'

(IV) e - CUB bx

» (j.) Let j} ~ tan x. Then _'V {O) == 0 .

We- n nd that:

") 2 ')

:v] := sec" -1'" == 1 + tall x==- 1 + ~v ".

This ~ri ves 'V t U}!:::: 1.

lei • 1 \

1, =:. 9 .. uv

. ' 2 6.5 .. ~ ... 1 "

This gives J\~ {O.) - O .

....

Il7

;"7 ~ 2 t l O:Vr..2j'., + 15y ,"'V 1 +- 6'V1vr: -1- fjv'V~). 'I'hi« ,gives y, .. ~n) ~ 27"i.~

.Il ~' ~ .... il . lu ~

. Thl1IR., for J! - tan x I we have

1LT (0 I-I,

.~ 1 '

'Y2 to} ~ o.

)'j 10) - 16

.. .r: \ .1

,J

1J lU~- n

.. I Fi \. ' ~

..

Hence, tile Maclaurin's expansion or I(xl ,== tane is [by using form til a (:3 }]

tan x

2 3 16 f) L 272 7

:f + 3 II x'+ ~ J'::' -r -7 I. .X .+ ' , .

I" ~1 .

(i i) Let '}j == tan - 1 (1 + x) ~ T'henv (O} ~ tall ~ 1 1 -:= rr/4.

We find th at:

1

v--~--

L J 1 + (1 + X)~

~~- 2· Thi~ grv('~<; .v 1 (Il) =: .~ .

2 + 2x + x. ~

~ 1 ".1 ~

YI) - - - "J ,) (~ r- 2x) = ~ 2 (1 +x) t2 + ':!_r of .r ")

"" 112 + ') . _L ~)~

~ ..... 1 IX '

2 .:2 "2 3J

~ 4 (] + x) (2 + 2_t +.\" J

_ tan - 1 (1 + x)~\V(~ have

,l,r ~n)::::;; 41t, ·U (0) -=.! 'v (0)::

~ .' 11 ,2 ' .. ~:~ -

1

-

2'

, (0' - 1 v. 0 ~ 0, v r 0) - -:1 and ~(l on

y : ~ J - !) J .' I \ l ... !i '

...

Hen ce thp Maclaurin '8 expansion of J' = tan - 1 (1 l- x)

,

]$

'J :~ ...
1 1 ~
tan - 1 (1 +x) IT 1 .. Y.' ,... ,x ,3 x
- ~ + -t - - 21+ . . - +
-
4 2' 2 I ':l 3 I ,5 I
,
~ Then)' (0)

1. 2'

r e

... i)

(1 + Ie"'")!'''

-.x 2 ' -,T

.1',2 -=- - e y + 2 e Y,1l1 -

- ~ '2 '

e l'U - 2vy-, ), TIllS ~_j'jves 11) (0) -= 0 .

,.. r'.' 1 ~ I~ :_ ~ I-

1 4'

. I (

""1 {O) ~ - Y l J)', =:: U,

.r 1 1 1 ~ 1 '

1. Di ftl:! re 11 ti a I Ca 1 cui U.'i

39

1 1

Y ~ ([)) = ~ H ~ J} ~ (O) = 0, YS to) - 4 '

0") ,I)
c·"ft. 1 1 1 ... t') 1 _1":'
9+ 4X - + - . 5' 1-
1 '~ 8 ;3 , 4
- (I' ..... • I • (iv) Let

fJX

.v == e cos bx - Then y (0) == I . Also,

2 . b 2 n/2 ax ill I

V :::::: (f} + '") e co g t'r. + It t a T~ ( b I a) ~

, n i

Thl~ gives

. 2 2 II 12 ( , - 1 )

Y T1 (0 l = (a -I- h) -COB" t~J II b I a

Hence, by -uHing the Maclaurin's theorem, we get

Note: If .' - (!US o. and b = s in U, the result proved above becomes

L cos na 1lJ'



n = 1

Examp] e 2. Obtn in: the Mac lau rin. '8 e .. xpau sions of the folloto ing [ur: cl ion s:

(i) edJD J

(iii) lug' see x

(iv) log (1 + sin x)

») (i) Let V == esin J; • TI1Pn.v (0,:;;: eO == 1 ~

\Vf2find that:

S11l L" rn' - .

_'V 1 - p (~()~ x - _y COS.\". I. ~'H::-. h'lV,(-\H

V_(O)== 1 .

• 'I

1" - v cos X - 2,' ~ in J: - V cos J. This givps v (to -= o .

. : I ~ _F ]. a

4·0

'V - V - (;OSl- _ 4v .~]n r _ Hv eo,1;..] X + 4 V sin x -1- V cos_r

LI r;. • 1- ' :~ - 2 ' 1 .

This gives v _ f 0);: H"

- • D

and .~u 011.

.

Thus, tor v == (/j~,n x , we have

'V (0)- 0

. .1 ~

Hence. ~TV Maclaurin's theorem, we aot

~- .fI ~ c ...,.l

2

. ,x

eto:iin J,.- == 1 + x +

2!

(fi) Let ~v ==. e~t cas x, Then 11 (0) == 1 .

.. ~JSO'l Log ~y == X cos z . Therefore, by' differentiation, we obtain:

~ -y

t' .., 1

,

,

- cos x - X lSl.ll .. t",.

or

'}I, = .V {cos .r - x sin J':). , 1

This gives ,1/ 1 (0) -= 1.

)'4 = ,~, 1 (cos .r - .r ain .r ) - _v {2 sin _r ~~ .r cos .r ).

This ,gi ves Y") (0) - 1_

'"'"

.)1.1 = J'2 (cos x - or sin x) - 2Yl (~ sin x + .r COB x)

- Jl {:i [n~ .r _ x ~1.n X)~ This gives 'V - (0) _. - 2. . ~,~

• 'J), •

.Y 4 - Y;i (e08 x - .to sm _1;) - d:Y 2 (~ srn X + X (~O~ x)

,- 'J ' 4 i

_ .iY:r {d C,)~ x - ,l sm X) + _v ( sin Jf +x cos -1").

This O'ive~ v (0) = 11 _

,~ , t1

and so o.n.

:r vus .r

V == e . '\-ve huvc

.... ~ .

v (0 ~ - 1 \' (01 -= 1 'V ~ 0 I = 1. \' (U J. =

- .r ~ ,I 1 • r:J J ':~

1 J .

J1

I) :~

,','''', ~ -'"

X l'OS 1" ,\ _ o.J _ .!L _ ii, 1

,~ , == 1--1 _1:+ ~. J

21 ~ a !

(iii) I~(~l .v - log' Hl'l' X. Thenv (0) == I.ng' 1 =: u . \\'c· find t~ -at :

sec v j ~ i I) X fl""l"

\I - - t_..:.. 111 JL; _ ' ,II,~ I T] V l~ ~

- 1 b

~Le .t

o.

v - 2 v v . TIt i ~ ai v ~ s .v:,~,' U) - u.

" ,i ... 1" '1 ~ l '\

.OJ

v, - 2 ')~~ ~ y IY;$)~ 1~hi8 gi ves "l~4 (n) - 2 .

.

Y (i - 2 (:-~\j; f· 4)' ~f + Y BY;j L Thi s gj vc-s J' fj to) =- Ill ..

a.nd So uu .

. ,Y (O~ -= o.

't! ;UI =- n

.' I ... J

fi J.'

6l

. . -

r(i, ) Let y - Iog (1 + s in J:) ~ Tlu-n j (0 = 0 . \V(~ find that:

·'os. _1[" • w •

v == 1 -.- . f h ~~ f!l v I~ v un z: 1.

w 1 l' ~1 n :\' - . 1

(

-

'I

~iu .r) (1 I siu .c) - cos" :r

~ - =-

. ~

t 1 -l- sin ')"

1



sru .r

r~q" r

[~l~S rtf, "Ivn~."'" \' 1 ~ I --

~ " .. ~

1 .

v, -

:1 ( 1

, I

si n . .r

i)

V - -rv + v v~. ThiH~h.~{'~" ~U).:....

• , • 'I 'I': I "'. l

• I

"'"'" .

!.I I ()) = 0 . Y J (n J - 1. y ~ dJ) = - 1. J':1 {O) - 1, v (0)- -~. v (0)- G

~ ] ~ r )

.:.J

H{~ncc~ l\1 a~la1J.rin',~ theorem gives

9 '1 d "l
JC"" ...... ."-
log (1 . .r 2 J:'- ... x
+ sm x) - x - - + - -+ :1 - + • 1 - •
-
2 l' 3. , 4 f fi ,
• 1 • • Example 3~ EX})(Jnd the [olloioing [unctions in, Maclaurin

,

series:

» (i) Let .v= log (1 + sm2 x) . Thcn v (0)·== O.

We find th.at:

, 'l

1. + SUl .r, which yields, on di(r~rpntiaUun.

or

,,\1 • I. '

V = t-~ c S] n 2.x .

··1

ThiR gives v 1 (0) == O.

This yields JI~, (D) = ,2 .

...

v - 2y V - 2 e - Y V cos 2x ~ 4: (1 - .1" si n 2x

~;) - -]' ~ ~ ']

~J

- - 2v Y - "\1. (v. + 'U .... ) - ,4y

, lL'~ "1 \..r ~ ~, 1 . 1

I")

- -)' (4 + 'V ~ + 3y ) _ T'Iiis vield ~ V ({), = o.

"1 ·1·~ ~ L3

~

t! = Y (4 ~ V I :3Yr~) if (211 V -+;:)V l. T11i8 P1ves }.' (0) = 20~

" ., - ~ .] ..:::' . l't· 2 . 3 ,0'", ,J .

and so on.

TllUS, for .v == log (1 + 8in2.it) .. we have

v (0 J - 0 v i: 0) - 0 - v ( 0 l -:- ~ ~I. {O) := () v. ~ O} - -~o ~

1 , 1 ~ "~ r , , ;~ • , 1 .

1. Differential (}alculus

1.1

HC!lCe, by Maclaur-in's The(H'l:ID, we obtain

'J

~"""

1 1 . I) '2,' ..1 _ '}'}

og ( + 8'111- x) == ,) I ..... ~

y •

J.~ 4: 2 f} tj

+-"=x- r+'--

4 1 G . .

r

(Ii) I.~pt ,Y == lug (] + c· ). Then y (0) == log 2 .

-,.' r .r - l' . . . v ~ 0 ~ - log 2 1

e' y - c ~ or }'l - e ... Th] ~ ~J V~.9 V. (In = e : = c - ,,).

~1 ' . '-' 'J' .

.....

L~ - l' l ~ rr I' . 1 (1 1) I

J' . ;;- [' . r I - V ) - v (1 - 1,' ). J 11 ~~:t I ve s ,I . r 0 J - - - - - 4' .

~ \" ~ I .., 1 - 1 b J:2" 2 2

("j Th ., 0 I

l" = "11 - 2Y"'lJ . I, "I;S glVC5 V l 1 =

• -'3 J 2 .' 1" 2 ~ : ~ . 4

1 4

o.

v - y" -

~ <1 '~1

I)

I) '""

'V

~:, CJ

-

1 This gives Y,I (0) = - 8 .

LI

v {O l = 10 e' 2 ~ )' ( n J - -.. v t U J =- - v • 0) = 0 }' ( 0) ;=

. ,_, 1 ~ , - ~ 4 l ~ :~ \ , - ~

Honcev Muclaurin's Theorem gjvcs

] 'J' 1 _,...,.4

I 1 :r J 2 1 J:..w <1.

~)g~ t + e ) ~ og : !- 2 .r + 4 - 2 r - 8 . 4 ~ r- ... •

Example 4. Obtoin the Maclaurin. series exponsion» 0/ the loll oto l} 19 1'1.11. ctions:

(i) tan

1 _l"

(ii) f/'r log' (1 + .Y)

,

( ~ li) (_1{J stu Ill"

I

'Ii.

» (i) Let. y == Lan 1 X _ Then )1 (0) =- 0 . 1

\V'l.!; find that )'1. =: 1/ ~ so thHL

1+- x~

(1 -1- J.2).y I

1

• • _ t i)

44

2_1CV _ ::::. 0

.' 1

. . . (it)

,)

( l -1- .r ~) V -I 2 (II + 1 J ,x v n fit -I- ]) v == 0

• f} I- 'J .' fJ + I ' . " n

... (iii)

y ~. 0 i -= O! V. (0 l - O. \' ( 0) = 0 ~ an, d R 0 OIL

.' 'l ,": G . , ../ ~ ,

Thus. f~n· J' ~ tall - I X, \V{: have

Y c (, n l - 4 1., v I 0 ) - 0 v , r n) = - (1 ~ _ '

~ ._ I,' .... 'oJ. ...

,) (1 I'

Hence. Maclauri n's tlleorpnl yiel ds

~~

" .l

- X - (2 !) _--,

. "J

·fJ •

.,.

,

((j !) ; f -I

~1 .'1

X x

-,~ ~ 13 +~." '

r . ,J

1"1'

I X

~ r _ •

7

'i i ) Let v == eX Jog (1 + x). Then ,v( 0) - o. Also,

1+ T'hi s yi f' ] ds -' I 'I to) == I .

v == L 1

+ x

. , . (1V)

~ 1 + x} v == e:!." +- (1 1 x) V

~ .' I '

J.

l! . .!

{1-1- viv -I- V 1, I ·

"f

(J ·1 .X rv ~ - X:V I - .r

1, , i

. , . (v)

1. Difft~rf!.ntia.l Calculu»

45

Next, differentjatl11g (v) n tinlL~ by using the Lcihniz's theorem, we obtain

(] + x) )' -1- in _ .1") \' 1

. n + ~ .s n I-

In+ ltv

\ r •. n

Thi:.; yi cl ds

y I~ ( 0) == - n y 1 ( 0) + (n. + 1) j' ( 0) + 1_

'"'n+L.. . n+ ~ - n

y 3 ({)):= 2 , y 4 ( 0) -=- 0., J' 5 ( 0) = 9 l ~ Y G ( 0, = - a51 a JU'~ so 0 n,

Hen ce J hy Macla urin 's th e( H"(~n1, we obtain

2 '~ 5 {)

x _ XC x,: X

eX log (1 + .c) =: 1 -I- X + 2! + 2 - 3 ! -I- 9 ' 5 ! - 35 - 6! I .,.

( ... )

'Ill,

-

L t e" sin

et .y==

l

X] I

. From t.his, we obtain =~

u,

and

? - 2. 2

(1 - x") y' c-_ _, (2n, ! l):r y - (11 -i rt. ) \1 == a.

/. t + ~ ~ n + 1 .T n

'I'husc give

v (0) = I, v ' 0) = a'·'.. (0) =

_. j • ] \ :0 J ~

Hence, by Maclaurin's theorem, we obtain

II 81n e

• I ~



Eli

xercisee

OblHin the Maclaurin's oxpuns.iuns of the foH.uwir-lg' Junctions:

I .. sinh L 2 .. cosh :r 3 .. ser J:
4,. . ~ 5~ ~ 6 J.-
sm .1 cos X ..
~: 1
~ -
-~ c': r;:_, l ~'It' r~, ·t I '--' j' ·-t' - - · J .) 1-) IL'. • -. 1, I·' ~l{ . I(.n ~ _~ 1::1~l Ion .:.-.~.;;_,. ~~j~,llllp· ._,

I'U,;:: t·

7.. l:"

.

" y ~.n r

"' f.

15,~ lug cos x

1 7 ~ I u r , I + ens t'}

18.. log tOrih x

19~ log- tsce .r 1- tan x)

· - I

2U.. S]I1 X

21 · - 1 .,

, ~ (stn .\")""'

29'

""!I

24~

Answers

j/' 7
x
- i I I
5 I 7 J
. x~ -1 _\_(;

3. 1 I 2! I 5 :! + 61 I)! I

I) 1 4 2, s

4·,~ .1'- - t- + - l: .. ~

:J . 45'-

1.~ 2 6

3X'- 45X ~

oi) 4
6~ x J,'''' X
- I- - +
~J 4 ~t8
.... [ 1'2 Jt '

7. e 1 - _:_ + -1 ' ..

2 , - 4'

, - .

,)1

2 t

91. 1 I X I X J- 4 . -' -+

- 3 .

1

2 X4
:r
~ '.
.. + .t. +
I} 8
""'"
X3 5,
io. .,.
.1:' -I - - 1
3 80
4 8·
121' 1 22 .r 24 x
- -~ ~ ~ . • -
4 I I'
, . 11 x

12 . ~ t· .

4 .

. 2

13.. x -l

'i 1 it

•" 1 J.. "L' d

~ r61&,{ r

r •

'::0:1 { -;.I IJ nl';' ( )} .r"
14,,- 'z: lU- -I- h- ) , la'il ~ L b
~ SIn 1'1. ~
11 , a

n=l
:l. ..:1 (i 1 ~) If}
151,., X .r - .t In", L.o! !I. IIJ
4r-:: + x - :\" .... F - -i
,
~ l~ 2' '~I -
11,
~ ~ ] l '~I t~
17 .. ling :2 .1 _T X
- - - . - ~
2 ? r 4 4 r ~) t'1
.... . L.I "} .
2 I. f; :l 5
l~ .. ~: - oJ X 16 .r 19~ :r lC'
- + -~ ,:r ~I r- , ,
'J r .i...J tf r r • ti 0). .
.... . 1 • "'"' -
~~ :r1~ 7
,") \' 1 :~ ~) 12 !) .) .r
2O .. , 1~ ~~ ... .' ;3 _. 5-
.. \~ -+ - I ~. -
'i I ". 5 r .....
. , . I . L Dihc~reJlt;al Ca[.r.."UlllS

.;17

,) ;r4 A G

21~ 'r .... -+ - -. - x + ...

• "J 41"'.:"

._) J

x3 .\ 5 .r: 7

2.2,'-. L I +

'~ J." r - + - -. ~

• .J r.=;; 1""'1'

~J ,J ,

~) :~ 4 .~
r'" X .r X
2:)~ 1 .. 7 fj
.., ~,. + - -- - + - - r
'J , ~J ~ 4 II fi 'r
"-' , . [ 2 .2 ~1 ·1

2A1. .(~ [,'["/:2.1 1 . t a.x . r l ·):r {Zi! + 2} ~ x

"'j ~ (-j ]I, - a:t" . - - - (J ~ I a- 1 ~ f a 11 4. 1 +

2 ! ml , ~

. . ]

1.8 IndeterJDina te fnrms

While attempting to evaluate certain limits, \\.'C of tell obtain

.. f·tl c a DO 0 00 0 liXI hi h

cxprPHSI011S 0 lC rorm 0 ~ ~ ~ . x 0Cl, [XJ - [J, ,Cd, W ]_]C .

do not j-epreserrt any real valu e. Such exprcseions are called iruleterm filiate forms.

Limits which lead to indeterminate forms (Ire genE)rall~y evaluated by employing a standard 'rule ](llO\V11 as L Hospital's rule, named after the French rnathemutician eLF.A. L'Hospital t 1661-1 704) _ This rule is 0 btain ed be low "

L'Ho'spitcll's Rule.

Suppose f(x) and g (.;() are [unctions such. that

(i) lim ftx) == 0' and lim g (x) == 0, orul (ii) l '(x) and g I(X)

x ... a

exist and g '(a.).# 0, Then

lim [(x) - lim t· ~lx)
- ~
g (X) 0' (~~)
X -7 a X~ a b provided. t 11.)(-; I in ii ton . the r.igh~t h UJ U-{ side exists.

- -

Here (I is any specified real number.

Pro of: Employing the Cauchy's Mean Value Theorem to thQ Junctions 11Jc) and g (_~) ill the interval [a:) x], \¥P. get

{(xl - f~(a,) _ t~(cl

g (x) ~I:f (0) l~ I(C) ..

Now', by what 113S been assu m ed , we have

... (1 )

((1) == lim tt:r) == Ul

x a

g (0) = Iim l~ (~'f) - 0

.~ ---4 0

T.ll" Et1J;!ill.l!p.rinJ: Ma.th·P!lIlutics·[1

Therefore, (1) becomes

IYx) ,E1. (x)

, _ , (2)

i1 J' ((d b ".#

Stl ~L' (I < r <... x, we have C ---1 (J a~ R- --1 0., Tliereforu taking [im iL~ OD both sides of (2), we gael

I, {tx) L' t" (c) ]' r~(x}

LIll ----- == 1111 --:: LIll

X > ~1 If (X) ~: ---;1 (I!! ' l C ) ".\' _, ts g r (X)

, . , (;3)

'I'his completes th~ proof.



o DO 1.8.1 Evaluation of limits oftbe form 0' oo

Suppose Iim {{xl == 0 and Jim If (x) = o. Then

X ---7 a

Itx)

1 i rn :;:::.

x -~ U g (x)

lim R (;r)

o 0-

.t' -t n.

--

J' -; il

Here the right-hand side is all indetcrminote /(JI"IIl, 1'0 evaluate th(l limit specified in, the left-hand Hide. we vmay PTIl ploy the L 'Hospitul '8 ru le given by (3.).

The rule (3) can be employed to evaluate

Jim

It.xl_ g (Xl

.

oven m

).; -t a

the case where /(x) --1 + l~~~ and g (x) ~ + DC! as X --1' a , Boca use, in th i~ case, we m ay (. -x pres ~ t h l!' Iim it in the form

L' t~·i~) I'

.Illl'!'_ = 1111

at r) II ] //. ~(' .r) II.

x -) 0 b \.; X -I 'l

o o

Thus, tho ru h~ (3) rll ay be employed whenever we encounter limits of the form 010 or + I';»/OQ, 111 TIIH11Y si tuul.ions it becomes necessary "to employ the rule r~peatpdLy until we arrive at a defini Lo value for the li 111]t,

J.' -. t)

(j i) iiII1

x > 0

( . ) I' tall :r I un

(iij) lim x t;lJH X-= I~,g!..! 1<1:)

x -) 0 )f~

(iv) lim x

.. ~. -1' 0

tan X

I, . tan J.-

J Hl - - -

1:':

x -. IJ

o n

:= ~m in x -) U

(/

~! {tau or) (J.."

(I

. ~ (x)

dx

== Iirn X - ) 0

2. ~f.:!C ~:rr

1

Sl~(·2 0

- -

1

(ii)

,x

lim a

Ix

"]1

o

l i n de t.{ i rill in a te)

o

_(~. 0

,x

= lug u - ~o,g IJ ==~If )'g (a.1 b) ~

x _C(_H';' ~ir -=-- 1 ~ )g_ (_1 -I x) _ u - 1 .

fi Il) Ii r11- '2 ~ -0 ,j n,n,~ ~ terrui n ute ~

.r -~, u ,x

.

1 ~ cos :r - l." .~ rn X

- un - - ~ -;~-I·

N "---" tl ..... 'i.

I ~~-i x) n

-

o

== ~ir.u

L· -~ 0

1 2-

(iv)' Iim

.r -~ D

.1.." - t.an x

- --

_rrl

o

.

o

.,

Th· 1, 1 ~ s e~:" \,' - lln~ - ~

~- oJ

~: -;) U a.r"·

o n

- lim

y __, (I

'J

""I ...

- L sec _x

-

n n

1 Li ~ l ~ I.'

r: . ,_, Sl'(' _\ . un :r L S1,.~l' _r

LUll - -

'~ 1

I. ~ • (l

1 :~



50

Example 2[0 Evaluate I he fhlloluin.g li111 (its:

~\.:. _ !l

(.) Ii e - Cl+x)

1 ·1111 . I _

X -? 0 x J og (1 +. &'\.7 )

.

X SIll X

(iv) lim

2 2 .

sec x - .-' tan x

. L·

e - c

(iii) lim ----

.r 'to U

J." ~ 2 log (1 + .r)

1 + C()S 4.1"'

» (I) lim _t ____, ()

p:b,- _ {1 + .;() 2 x log(l -I- x)

~ (indeterminate)

- lim

x-, 0

2 (1 + x}

o o

log t l } x)+ x/(1+_,·)

4 e2."l - 2 4- - ~

{ I} J 1 } 1+1- I

(1 +x) + ll1-1 x}';/. .

··Zx· 2.22- O.

11· n 1 SIll . ~. + .' Sill X ~.l .'. S111 X = 0

(i i) - L. (indeterrninate)

x ) 0 cos x - cos x

- linl

\ 0

_ 1- ~ 2 C OL~ .2x + 2 Sill 2.1: - ~ cos x

- Iro . .

- sm :t: + SIn 2x

n a

r . -.4 sin 2.'r + 4 (:us 2't -I- 2 sin x 4

-Lm ens x..J ~2 COC! qx -.

l' --) 0 - L"'~ ,-',. Q k.'

(iii)

linl x )0 0

x -x

e - e 2 L~ (I -1- x)

~

X SIll. X

o o

1. - r 2/( [ a
1]111 e + e +. _r}
- -
. 0
" lO X C08X I· Sill .r
-'I': - ,\' 21( l + 2 2
(J - e :t' )
- lim -
- " -
.~~ -}.. () X HHl .t· + 9 L::n~ x 2
.~ ]

(Iv)

Iirn

.r -) n::/·..;t

2

~ l!t~ .X 2 tall X-

l + cos 4x

o o

I"l q

') .:. t .J..L.

1.. ~ ~eex .anr - .u sec J ..

- lID r

S1.U 4_,-

, l
1 tKP(~ .... x~ (taB x 1 ) 0
-
- Iirn --
~ I' .r.r ,~ si 11 4:r U
~ 1 2

51

lim

~ ) }"L/4

(2 ~e~~ J[ tun x ~ {tal} x 4 cos ·1t

1) ·1

1 0 + (\r2 ).:1 1 ~.
-
~) - 4: :2
' .... Example 3. Eualuute the [ollcnoing limits:

. . -]

1. -, sUJ. x 8TH J':-

(1) .im . -9---

r ---') {) x .....

- . ~1

( i .i 1'1' ) J"" 1 0 g (] + x )

1]11 - ~ 3

x ) 0 6111.r

» W'Cflld that:

~ . - 1

(i) 1. SIn. x SIn x

'\l,· un

")

.1 ----t 0 x ....

(ii) lim

x~ 0

sinh X - f;i 11 J[ . ~

:t sm x

( ~ ''') '1"' sinh .r - sin x

1 tJ ' '[111.

L}

. _., '1.1. "_

.\ ---4 0 ,;'1[ S ,I, n x

(iv) J11'11 X -t 0

tall x - .x 2

x tan x

11", [cin x

Jll1n - -.

-;',. ---4 0, x

-~ , ..

I le,

~'[

( . J ( ] J
lim SIn .r lim ,"·an ,.'t

t---ri' 0 X .r ---;t, 0 .r - 1 ()
I_ lim SIn .r
.1. ---!I' 0 X 0 lim x ----) 0

~ x

. 2 SHl J.:

sinh .r- si 11 .x 11

\7'

I 1

--

J lim r

f -~ 0 \

~

"

-

J. ,lj'in h .1. - SU1:r

.un :~

'I '" 0 .1

- 1

o o

- lira

\ ---f 0

{OO;:: 11 r - cns x

o o

·1

,~, ..::.

-,J.1:

Ii in ~. . 0

~i n h "- ,~i n f

1. cosh x I

Inl -- ---

'I' ~ 0 fj

cos x

-

-

1 3

.~

I n g j)1 1- X')

(iii) lirn

,\,0 .' u

o : ~

~ 1[1 .L

o o

( ;~

III. X

~nll -

• :1-

I s II Sill' X

I i In

;l_' " 0

~ . .'1

L,Og ( 1 + oX )

o n

- 1

]11'11

]

r

-

,., '2

( I + .r' ). ~ Ji

(Iv) Urn

t a,ll J.t ...... J: tun x

o o

I i HI ( " X

n. , tall ,.

~I !J ~ P

dnx - .r J ;~

x

1,"' tanr - .r 0

JUU :J - 0

!,O _t"

:2 1 n
sec x -
- 1 llm -
,) 0
U il3 II ..
.~ .. Ii. '.'- l' 0 0

o o

1

_I

':_III ~)I

Exanlpl Jo, 4 .. , }~ilul the ronstant» a atul. h .... '0 'huf .r (1 (I co S .;y) - b s i n . ...: = 1 0

lil11 3

x I U :r

I" .r ( ]

un

; Il

U COH .~:) ~l \.

{) n

]' .X l

:: IU} -

l' ~ ~

h COR X

1-1- (I o

b

I.

snv ,~I . )

,

Rvidpnt.ly. if 1 r- u - h,. 1 hen I = r. • Bu 'I~ f has to be nqua l

to 1. Th ere; f<U'lll wr- tuki: 1 + (f b == 0., ~n thai (i) i~ an

indeterminate and ~""le proceed furf her,

For 1 + (1, - b = 0, L ., b:: 1 + as wu }!t!t~ frnrn (i),

1 ,::

.r ~ - H sin .r ~ 1° (1 1 Q COB .r} ~ t J 1 i HI

(I J cos .r

n J

. 2 • _It"

- ]in1 r - u

,

a :sln .r - ax cos x - fL ~ Ul. ~l"

fix

t I +a) ~in:r

o U

- lim

• 0

.

a .:.\' stn x - n cos X + ( 1 - (1.l ("Os x

n

1- 2a 6

Th . rufuro, 1:= 1 b == J r- [1 - - ~)·/2 .

if 1 - 2CJ ~ G, [) r 11 = .- ~1,/2 .

<

Thpl1

Till s. a = - 5/2 .and b - - ;1/.2 a re th« 'V" lues of (I and b for

which the giv(~n li mit i~ cquul to 1, •

11 ~

(1"') I' ug HI 1 X

irn

)r +» 0 cot x

(i.i ) I. i 111 log t(_111 X

n x r . itA

» (1) li rn

.t: ~ 0

II '

109' stn x

..JP _

cnl, J(

('U~ J:

Ij

cosec" .r

Co)

'J

- eu~ol~C~ x

- lim I)

II oj el ".~If.:. .. C -.1 l'U I X

I.

1 i III lau_y

o

C"li) lim

:It: ! U

lUi! tunr

_\0

ii m

" ~ 0

Jog 'tan ,~r _ ..

log .r (,.

'J
- ~an.l' ~
!'r;l't; .r I _r ,
~ i ~ II Inn ~
~l {' ,t
. II 11 X \ I I > f,:L f] r li4

(
liln l .; X I' lin) ~ l im • J 1
- I ~
- ~C' J. - H\:.Jt~ 1C~ .1
- .
X -~ U j at -; ~1 x__" 0 E .

X irctses

( j) ~!n

r ; U

1.

cos X

(~) ) I~. • - ] ~ g (1 + X)

~ tJl]j.J.

I - rns X

~.

(~a~) lim _((.,'

'I. - 0

_

S1 n Jt"

(I) ) lim ] cos X

l • n J.: .sUI X
:, I ;l:r - 4
(7 ) hrn .r
')
r 1 III ..... X - ;i
'" , .... .'(
,
(9) lim r- srn r
~ -
"
r It r) r '/ (~) , . cosh X - cu~.x

~ J IJl11 •

.\' -l! 0 X s] n ~ .

J" sinhr - log ( 1 + xl

I'R u n ~----.---':":;"__:_-__';"

X ':-:111 _'_

.r • 0'"

2x - i~ X

----

? .

X~ SJn.(

l' tan x x

1111, ')I

1 --j' 0 X- Lan x

( 1 f) 'l Iim

r • n

,2

f-' ~l r'O:-l x - 2

. :'\

.\- stu x

( 10) Iim

1 -4 n

.J

tan" l1.."

(17) li 111

_~ --; 0

· , I :.!:

f:H n -T t-i ~ 11 X - X

r20~ lin'

t -. I)

ctd, x - l/x .r

2'~, 1 v;~ lila .~~ tile fhU nw i ng Ii m iI ~ s:

)og' .r

, I' lim

Ing-(Jf rr/~)

tan .r

(,1) li In

sec Tl1'

t ~ H1 ;i:r~x

t tl ~)

,n ~,

~.'~:J

] og (.1' - ((.) log lc-t fl')

. o~ I.'. tall :Jx l n) 1[[1

J; 10 rU~ tarr x

(7) HIll

cot 2,r (9) Iirn -

_~ _I 0 (~(ll. :r

lim

.1. -! 0

~rin Lbr + a ~in x a

_1"

is finite!

110

Also, find the limit,

4\,~ In each of the following cases, find a and b:

J' ]' _t. (1

,1) I III

x ___,. 0

n CO S x) + b sin oX J

J"r

1

-

~3

. . M X (2 -I- (1 (~O ~ x) - h si 11 I

fii) lim

,5:

x -} n x

l

-

-I ....

. ;:)

Answers

l~ { 1 1 1/2 (2) 1 (3) ;1,/2
(6) 1 (7) 6/5 (8) 2
i It} 1/~1 ( 12), 1/.1, ( ] a ) l./?
(16) 1/2 (17l 1/18 (18) 2
2- (, 1) 0 (2) a . 0", ) o
. ~ { ;.j
lG) 3 ~7} 1 (8) 1/3 (4) 1 (9) .l It} (14) liB

~' 19) 1/2 (20) - 1/.~

(5 ) ]/~
(1.D) - 1/:1
( ],5) 1 '1 r)
/ ..... (4) 0

(~r) 1/2

15) 0

3 ~ a - - ~: li 1ll i lJ - - 1

4. (j) a = 1 /:2 ~ b =. - 1/2

1~8.2 Evaluation of limits of the form 0 x DOl OG - PO

The L' Hospital's rule may also be employed t.o cvnluatc the limits wh ich ~rp of "the forrns 0 X tx) .rn d '-Xi - G-a ~ For this purpose, W·Q fi ri~t rr\\lrit(l l.hu functions in \'0"1 vr«] in such a way th at the limit is of the form 0/0 or ee/ co; GIld. tlH~~n W(I employ the .L'1 Husnita l's T'H ,Ie_

(i) H rn x Io.~ j_'

(1 i) I, i m

_] t hI

l ) .v

_1' -,' n .1 -) ..

( iii l Iim .,~] og La ~ i _y \ ~ i,)

56

T~ [I, Et1Hi1Zp.erin{! Ala th em h I i{'s&ll

», (l) lim X log X == 0 X - 0(;1 .\: ---7 0

lug _r - 0<,

== Iinl

1 ~_\-

,\. c.:<_J

_l l n

== ~ 1111

_t -< 0

, I •

[ 11 XU )

I j nJ~. X) == a

:r } 0

( i i ) li 111 to I t

1) .r == Ox 0Cl

] / J.- 1 t 1
Iim a lim a
-
)_'----.; l/x I -) 0 l
roC' - lim

t -, 0

i

(I t log (I)

1

liy J / Hospi tul's rule

(~ 1

= (1 ( 1 n g u. l == u g (1

(iii) \ im )[ It ~~ lan x -== 0 x - (.;,QI

X \~ 0

lo~ tun .r I. 'Ll-

-:"'

=- lim

".;I

] . ,~ec"'" _'('_. tan x

- w'i~r' - -

. ~ ') ~

( 1 ,I _ ....

I. --; I _t

i

II iu I J 1 ~ 111 (or ~el: x) ==

~ - l (I 1\ tan X ) _\ ,[I

o ~ 0,

. .~' [1

(I) J1J11 .~

• (J J.:

)_. -,

( • ~ ) J'. [] I ng ( 1 + x) 1

11 . 1111 - ')

~ :t I.} .r rr-'-'

C' ) lir r J

tv ,In 1 I -.)"-

,~ ~ u x

1 ]

.,

sin" ,\"

.'-

, J1 .)

»

(~) 1-. [ 1

1. JIn ~ -

x~.O x

1

j=

y J n
rJ - - 1"
~ 11111
0 (.(:'1 _l 1 ) 0
J I X -
;0; l
,.0 ~ 1
~ lim
t) 'I J -I,." 1
l ~ l.l' ~ ," - ( ~ ~) _l.

_t + ~ e

- lim

un ~i I'm .r -10

[_1 _

x

1 2

lug I( 1 +. _t:) ] 2

X

x - ~ og·. (1 I .t ~ ~ 1 i 111 :____

2

.1(. -1;. 0 JC

o o

L - l/t1 I J_') ~ Iirn

__ --1!0 0

( . '" . ) 11.l,

]i111

(sec x

tan ~r) = ~<l - "r"""

~ - .q~ll. X (

--

(·0:-: .r n

lim cn.~ .r
.
, ;r .'~ • ...:Ul ~:
-
lim cot .~ - 0
-
\ ~.!' 'ill ~ (li,,. i Hut

.r , 0

[ l:!_

.r

~_!_ -- )

, ~)

HUl ~ _\~

,

- Jim

• j 0

,) ;!

;-;1 n ~' .~

••

,~Ig .JI~

,~ I ¥~ .) )
lim r .r. S11J ;t- -\'~
~
:l q
, I' r), ~Hl ,:r .r
....
(- ~2 . :l '_J
X stn ~r :r~
-
l l i In lim •
~
- jl
~ , 0 fi;1 n ."t" r , 0 X
I 1

-"2: 2

I~ sui _( - \"'

run! J

,,-. 0 x _

o o

') n

- , :-;] n Lx - LX

Iim 'I -- ~ by 1./ Hospital's .r ule

t !. (J 4x"

1.1'~-11;' 2 ~Os 2.~ - 2

~ i !) by L' Hospital's rule

,~ · 0 12._'t·,~

- """"-

1 :2 . 2x'
lirn - ..... StU
(, ~: ~
~,' ~. 0
1 1 im 2 cos 2_.y
,
If) 1 jo
n
,\ ~.,I' :2 n

1 3



E ~;

xeretses

( 1) lim (1. - Hin _t'} ta n ,lJl

,. .J,o 'l(/2

l:l) H m (s,ec ~~. )1 ~,![Ig "l'

. iii" oj, 1 , ,~t,

, l '2 1]

(4 ) Jt~n ~ - -~ _ ]

,# Y 11 :r~1 ,.1

I

(J':,' 1'- l ]

,i1) un , L ~

'~ ---"} 2 .x - ,2

~,

(9) li m ( COSI~C JL- - ,~,nt ,\' 1

,'- 1 0

f 1. ] '\
~ 1 1 ~ Ii m I
- -
,
~ 1) \ .r S'lI1 .1
-. }-
( ] I,
1 "~J ) Ii rn '.1'
~ ~) ~ln1 u 1;, -
I ',I
~ - " J ~I r .,
, .. ~ ( 12 ) i im l' n - cut x )

r a

,~ -I 0 -

- 4 i : r 1 ~ 1 _ l ~ un, :2

i ~ U \

1. IJi/f(lreuliul C .. 'atculu s

59

An,Sl.vel·S
I 1) 0 t2.) 2/rr
{7) 1/2 Uj) 0
,.
(13 ) - ~/:3 (14J 1. /:j (3) a

(r;) - 1/2

( (1) - I ~1.2) 0

8 3 E 1- t- ru -t f tb.e f 0.0 0 100

1 .. !!If . va ua Ion 0 rmt s oftl e (Ol~l. ,00, .

'I'he limits which are of the 'fOCDl 0° , ~ 0 and 1 ~ may be evaluated by ruducing t11l31TI to the form DIU or co<:,/ IX) , For this purpuse, we first t.ake the logarithm of the function whusc 'limit is required and use the resul t:

If L = lim 4' (x), then. log L == lin') I log lP (J:) : .

The following examples illustrute the procedure.

Example 1 .. Evaluate the [ollouiing limits:

(i) lim :C

_);; ---;, 0

( )tH[1 _T

•• I! l'

(II) . 11m. -.

x

x-~· U .

(i'ij) Iirn

cut .r (sec x)

:c ---4 n.' 2

» (i) Let L::;:: 1 LIn .r\:== 0° x ----? 0

TIlcn

,.

iog L == lim _y log ~\~ .r -I 0

o x - DO

= o·

~

see Exam ple I. Sp.c1.l on 1.8.2

o Therefore, the required limit is l~:= e :::: 1 .

( - J.tnn J.-

(li) Let L == lim _! == {;.}l

X ---:t 0 \ ~'\-

( 1 ...

log L == li m tan x log I -~ I == 0 X I.;.{I

.r ~ 0 \ X ...

'I -. U

~ng .r cui x

- Iim

fi(J

T. [1". Engineering M'alhenlalics-ll

= l im

_'I. -) 0

- I/x

by 1) Hnspi La 1'8 rule

- ----

'-;I

- C08t'C .x

, ~ t I 2 J

l' S111 x l' .. l Ern _\'

--: ] Tl] _ - Ull ~) - - :r

y , u J l ---i> {) ~-~

I )
r tim SIll :r r hm 1x 0 O_
j' -
- -
J -,:.1 0 '\~ r a
-,
, o Therefore, the ceq uirud limit is L == e == ] .

cot r 0

(iii) Let IJ ::; lim (ReC ~r):; ~

x~ n/2

'I'hcn, lug /JJ ~ lim (cot .r) (log sec .1') == - 0 X Qo .r -1 :iil2

l' lag sec _r

- l]nl -

l --', 1t (~ tan x

x --j, :ru~

tan ,r iJ sec" x

by L' Hospital's rule

= lim

== lim

-

SIn x cos. x

- 0,

r ) [[:-:2

Therefore ~ the rcq uired J imit is .L == eO = I .

Example 2.. Evaluate the [ollouiing limits:

( i ) ii m (1 -1- x} 1 /~' ,l~' 0

1.1/X

[ ):" . x .'~

( .. ), I' .Q +b.· +c

11. 1m

x~ 0 3

.

I. .. ~ - ) I' (t ) tan 2~

,]'11 im .. (1) x

_\ ~J :IT ... 4

(iv) lim (1 ~ sin xY::()( x :c~o

» (i) II l I·

o 1·' Let .~ - 11TI

,r ~ a

(1 + x)1/x

1 0

log L == I i III - log (1 _, x) ==

x-,~Ox' 0

J11.11 J.: ,D

1/(1 +.t:)

1 I "I

by L' Hospil.al's rule

- 1

1. Diffe,.enti.u I Calculu»

61

Therefore, the 'req uired limit is L == e1 == e _

. (01' + I_J '_' _+_{'_)" '11! .r __ 1 [)o'J

(ii) Let L ;;; Iirn .

• 1,' -, U 3 }

I"'rhr.:n~

1 10F! ( 0.1,: + b:t + .r 1
log L lim C
-
-
1." -:, 0 .·r 3
\ ! og- (0 -' DO' \" n
ljrn r + c J - log a
~ _ ~
-
r-----O .t" n .r >- 0

by L' Hospital's Tule ~ Jog a + log b + log (1 _ .2 lOfT, abc

1+1+1 -3 h

1/~

== log' (fIhc) u

Therefore 'I tile requir-ed Iimi tis L ~ (abc) l!:i .

(ifi) Let L

lim (tan X)tan 2x = 1 no x ......., 71"/4

Then, 10g L == Iim (tan 2.x) leg (tall x):: oo X 0 .x --1 f[/4

1', log rtan J.:) 0

un In _,

'"T. '~T!' r4 cot 2x 0

_~ • ,'I, /

- lim

... _) 'r.;: "'4

:2

sec x/t.an x

:2 l

- 2 cosec . 2.-r .

]

lim

2 sec x

------~.

i)

t a in .1' eose c .... 2:r

1 2

,~ ::::-1 . ] .

2

.l -4 :rf /r~

Therefore, the required limit is L = e - 1 = 1 e

(iv) Let L ~ Iiru (]

x ~ a

sin .r l'ot _t

Th in lug L = lim (cut J:.~ log (1 X-I' n

t- lu~ r 1 sin '()

un

n tun x

o

- -

n

rco~ XI Ij 1 - li~u

\ -.

if

hy L' Hosprtal s rule

~J cos x

J T sin r

]

I

e = e .

il\:

f!!) 15 .( sin J: ) -

11\1:1 :un

.1' ___" 0 X

-f

II.\'

I ~.' 111 I :. . (t an , ( 1 IljFl rm

.r ) 0 x

. 1 I,"'J;.-

,).). (l) L t L == lim ( 511~ X J 1 P'I

X ) 0 x

Then, log L = lin' _r -, 0

1 ~ 0 g ( sin x J == r)(J x 0 .X

Now" we recall that

.

Sln _\"

a .J.

x' .x-

- :r- -+ --

:1 ~ .~ ~

. ,

Therefcn .

. ~ I
U1X l J; :f
:J ~ 5 I
1: , . log { '} .l.t ).
( Sill x ) ~ I
, J[ .r
arv a - I -
.,. .r 3~ G~ ,
' . ' .
log 1,1 ( I t'l t:J I
== , I ) - ~ I -
~ ;)
~. 'i
t. .r J:"
- - - -~-
--
. , f:
f _ ,J • 1. Dif{erent,ia' Caleu Ius

1 1 l Sl n x J X' ~ I 1 h - J r

- o,g - .= - ,'), 1 + termswithh ig let p{l'A1C'r~~] x,

x X, Ij -

USi11g thi S r we get

1 (" sin x j'

logL == lim -log = 0,

x ---t 0 J: .X

Therefore, the required li mit is L == eO:::: 1"

(Ii) Let

:1

, l/x

L 1', [La.n x J 10<.-;

== .l Ifl ==

.x -) 0 X

..

Tben~

,~ " · , J] , [ ta'l1 J( J' 'l

logL == Iim .1, .2 log , J '

."--40lX .. x '

'We recall that the Maclaurin's expansion of tan x is (see Example 1, Section 1,7)

]:-1 2 5

tan x = x + -3 x ~I' 1r.::..t t-' I

. n

so thai

where

( tan .1' ). ( ].2.

log .x .::;; log .. 1, ::I x +

( ('2 ('3'

- log (1·+ 1):;;- t -2 -I :.3

1 22 4

t - r) T'-' + 1 5~ x +

rJJI .

, )

Therefore,

1 1 o g_. (. tax',n,'r' X)'. -_ ill]', + . .

J t.errns containing x

~

.1(

Th' ,

"IS grvus

l L 1" J 1] (tan .x ) 1 1

109 r , = ,llll. I ~,og. ~ == - ..

X--"7 Ot.x ,:\ J 3

Th tl . d li it i 1.". = e] /.'3 _

. .us,· lC req Ulr8 llTIl IS ~

Exercises



Eva'i uato ~,11 p fn 11 o'\-v'ing J. i ini l!S:

(1) lim x~m I .r -..., 0

( I) I l i ll1 [cos J.~ )il:'rl~ I

r -10 n :l

64

( ,3 ~ l.i. m ( co t x J ~ U 1 , _I." -f (~

f,t} lnu ~eotX)m lugt

1 • n

{ .Ill) h wn (fl \..!_ r "I -

..t -~ l'

)1

fl

.L - ,

.r

f 7) lj III ,I x ·u

" . ')

( ./ _ . I,: ,I rJ 1 11' , .. ( n

l ~ ;,)

1 .' II

C I' . 1:lIL \

( .) ) Ul1 iJ51 n X I '

.r ~ n:/'t

• i>o1 1

~ 1 U ~ lirn (cos _1' J

_I.. • n

(11} lim

:r -~ 0

"" !.

(~I~)'~' )11f

.1.1 "'-' ) '~ I n I (tn."::.l

" • (J

J ~ \'

(] 4) Ii n11 ( tn~ _.~' )l

1 • n x

~

,! ~1 y

( I G) lim l S~, n X J

' r

.1. -), 0 -

,~

. .I/r""

[,i' l' ( sinh x 1

~ ,1 J UU

'~f

l I 0 .'

Answer:."

t ~ j 1 (2) 1 (,~) 1. l i~') l/~~ (fd i fi 1 u'
(U! r'
-~ if ( 1 1 ~ -, , t~ ~ 1. ("'./[-'
(7) e (~) e (~ , 1 t lO} 1 t/"l~'
. ~ I ;], ,e12 lltl) ] (15) 1 "[j ( lJi J 1 . t_j
1 (' r 1.9 Der-ivattves of arc length

In this a n d. the next section. we consider some geometrical npplicatiuns of Difforentiat Calculusv Tndced ~ the lU,3'ttel'" contained ill these two sections is n cont.in u ut.ion of that prese-nted in Sut::t[nns 2.:31 and 2 .. 4 uf Part-I.

Consider a curve (.If in the .ry-plan '! Let A be a UX(1d point

and P be a variable point 01]. C,I. SUI}PU~l~ WP nH2a~UI'U thp distance of P from ,A nlong (_~) _ 'Let s he th]~ distance: then s is called the are len R-fl1,ul P'. Let the b~,h,n,gIe~Tt to I r al I_) 11!1 (:; etthe X-f1X~.s at the point '1'. 'We denote the allgJll hetwcun j:JT and the .v-axis hy \1:1 ~ as dune cnrlier. 1'IH~r~ (s ~ 'it) are called th.'p ilttri.,'I",~',i' OJ eoordisra ies of 'l h(~ pu~ nit 1 J a nld we VIo' rite

1) ~ s. \~I), or P (, .... , ~.q hl mdicnt.e t.hi ~ fact.

1. Ditf"erpn fiu I ('a.l(~ullt."l

"1/ ....

t;S

- ···.r

N ext, consider H point Q on C in a neighbourhood of P; see F j gure 1.9. I Jet s + (),~ be the arc length. at Q, so that 6s is t.11P distance from P to Q measured along C'. Let the actual distance between P and 'l be &. (111131. ix, 0(" is the length of the chord IJQ). Then 8s == OC ill the limit as Q ----)P along C. Thus,

For further analysis, ~vp conaider t.h~ cuscs of Cartesian and pol ar curves separately.

1.9~1 Case, of' C,artesian CUr\Tes

J .Jpt t he C artcs ian equation of C be .'~ = r (_"r) and the Ca r tesi i::U1. coordinates of P H.11d Q be {x, )i} and {,':r.+ at.. .Y -of ~v) respectively. Then, by the distance forrnu .. la, \V{~ havn (SC(~ Figure 1,,10)

66

rrh . crefore "!

:2

L+ r~J' J

c,\:

(-.

ur

. ~. 1)

We no ,~~ 'hat S·x -) 10 as Q~' _ along (~~ Therefcre, in the limit as fu: ---? O we have ~~ = 1.·" Hence, by taking the limits on both rides of (1) as 0.1; ~ 0 \"1" g·..:It

i) t1l :..
-
,(is 1 -I- (d~ J ... (2)
~l dx I
f ::t: 'I'his is the form.ula fOT the d srioatlue (or thedif/elY!'ntial cocfficient ) ofthe arc length. S LV. r. t. x.

Similar-ly, we obtain the following formula fori the derinoiive o] s io.r.t. :v :

\ \/2

r

SOlne consequen -f!!:J~

(1) Suppose the equation uf C i . Ffi ven ill thr par. metric form -~ = x (t) t Jl = _v (t) ~ Then expression (2) becomes

cls = { 1 d.x

1'llereforl1.~

") ...

l~ dv r dt. ) t./XI rl f

"J i) 1 I .
.J ( tl.x ... -
(Is (is }'( ) ( dv )
L .. I I I ~ 4 ~
- - • - - tl 1- :~it I
--
e.if. dx ,r/'t ' Ii This is he expressio 1 for the rlcr;vulilJ.e of'tht;! (l1~("/I}H!1th "H}.I".I. th {' I) {I nu JUi! te r t I

67

we get

r df<

~ dx )

( (hoi II

t rly J -

.) 1 /r)

( 1 -I- j n H - l~/) ~ - H ec \jI

_ , , Ui}

{ 1 L t2. ) 11 ~ , ,

-I eo. ~~.! , ,= cosec ~J

... (6)

Th(1se .y j uld,

dx ds

dJ~ ds

_

srn '\~'

, .. (8)

(I J2 (I J~

(,X ,()r

- -I- - -

tis , ds "

",:1 '2 . 2

(d.s) =- idx; =1- (dy'J

1

.. ~ (g)

... (10)

Here ds i s ~alledtJl e nrc elem en t (If (I at th p 'poi nt /1.

The following Figure exhibits the geometrical connections among dx; ely, (is and '+',

P d. ,-

{1-'iJ=! ure 1 .. 1]

(Is iI'S

Example 1.. Fin t/ tJ nd

ax (i",/

(i)

OJ "!

..... T .. )

n

1"1~ l I. ,~"ngi.lle~riu.g ll-la tbematiee-Il

» (:i)

we have 2ay dy == :-Ix2 (Ix

( r_i_y 2 ,,~
1 9x Hx
Ot - - -
- 2 2
dx i.lll 4a
) ,_ ..
\ "'" l ~~ r r \12 - ( {I+(~~r

Tl1C rc rj) re,

d's dx

ds

--

dy -

l/'d

, ~lx,· J

1 +-

40':

11j~

( 417 J1/2

f :::: 1 I -, "_',, ::;;:::

~x

_,2/3]1/2 ,1,(1

--

8 2~",1 Iy

,

~ '.

, 2/3 21;}

(i i) For the curve x + .Y

,2 - 1/3 2 - li:~ tiy

3, .X --I 3 .Y ;;; - 0.;

Therefore,

~ J 1]-

s: J,:)

== a

we have

,

or

rly d'f

( J1/3

J~ x

ds _ {l + ('-dLYx'~ l'~ } U2 - die

ds dy

{, ( ax j~2'

- ] +- -

dv

- ..

curues .'

j·V2/3 lIn 1 + 2/~}

_.r I

1 2/3, 2/3111 :l

_ X 1 l'

'il ~ .r

1/3 n

1/;J • x

II -''2

r

1 ''1

{J, r ,)



11 3 Y

ds ds dt ' dx

d. If"

u")

and

lor the tollolving

( i ) x == a (t + s j II t} ij .Y = a ( 1 - C 0 ~ t)

(j i) x ~ a {cos I + t s in t) .~ .Y -= Q (Sill t - t cos t)

» ('i) From the give 11 CqlL3 tion s. W'P get

dx

- - a (1 ~ cos t )

tit . ' j

dv

,- - a ;:1] n f

(. t

d.J'

-,

til

(it a sin r l

dx - a { I + eus I) - lu n ~

SO th: .t " = tl2 ~ onsequen ~ " [by ormulas 'Uid, .(ij,) and (il)J

dis t

:= s ec ~J :;;: (Ix

(18 dv

L'

t '_ cosec \.1.' == cosec

T 2

ds tit

{( ~~ i !) I in
( dv J I 1/2
2 0) 2 'I
1 ' ~ I
+ .' J - l(l I cos t + a srn t I
cIt , )1/2-

" - 1 J J! r- l- ~ t

- a (2 + ~ C()8 t) == a Vi 1- ,,2 C UH -

I}

......

(H) From th given equations, WP gel

rlv 0

-~- - at sIn t

dt

Therefore,

rll. (ix

(It S~JJ ~ (I f cus l

- tan t

w - ds d_t;

set: W = sec t ~

cos c t



Exerciees

-,0 - ds 1

1.. 111nd I (il I" t te fo llowi ng ell rves. (,,1;

[i) y = c cosh fx/(") (ii) y:' - 4rtJ~

2 1;,0 ..J ds (' Ll "~1 .

,., _l4 mn -,I 0]" l.- Ie ,U,Ol owing r u rves :

()~

oj I ~.- ~ I

I_I,

( iii ~ ~I' - ~rl .r

3. "j nd :~~ m' h€' Iflll!)w ing curves :

( i \ x n t'O~ f. v ~ h Ki u I ~ j j) .r == [I (l - .~j 11 I..J Y ~ (l (1 ~ ~'U~ ~)

t • j

n i j ~ .1 - 0.-1 ' ~ III t ~ Y = ( r l' ' en ~ t

• ~~ L ~ ::1

(I'll x =- u c·os t, .v =- o ~U1 !

(j i )

r II 2

fi 1 f ..

X

~ f

(iii}

') .)

I .&J -I x'"

--~

') 2 n .... - X

I "'~

(' ~ I I i) I ,. ') 1· ~ 4 1 i '2

. H) I I cc J S cc 1" Lr . ('] r ( 11..1 ( + Y /. a J

t iii 1 ~12 a e t

2 2 2. :2 .1/2

(i v j :l sin I cn ~9 t (u cos 1" iJ . sa n t ~

19· 2 C····-

.~ . .. ·as

SUPI)U:· the curve G' iH m·\lcn 111 tile polar form with r == f (el as. its equation r I ~pt t.11e pul: U4 coordin at ,8 of' file points Pi an.d Q be (I", (1) and (r""", fir - 0 + 09) respeetivel .. y. . 'TOp the perpendieular PN from I' onto OQ (see m~ igur .. 1.,12" . e find that

PIV = (J I sin oH ~. OF (B~) == r 00

rlQ ~ O'IQ (J f == ()QI ~ UP (ct 8 loB· - OQ - OP

r == or.

Heru we have used tho approximacions sin 159 ;;;:: ~O and

'Cos 60 % 1. (Sa OCill,g sm a 11 (because Q is taken in a

neighbourhood of IJ on Cl .

~ ~ cnco from II p r~gllht-:. .ul:-Jed lri,an"glc PNQ w have

P(~;!. ]JAr2. + /\lQ2

or

01'

:... ,]/~.

( §r J !.

. ()f) J

I.._

1. Differential Calculus

71

OS 09

os

_.

oe

2 ,1/:2

&; _ 38 {.~~ (or ~I l

-,- - -;:;- , + j'

B8 be HA

, )

. ' .. , 1)

We note that b8 ---7 0 as (J ~ P a l~lng (: ,. 'I'herufnrc, in the limit as ae --4 0, we have ~ = 1. Hence. by taking the limits on both sides of (1) as 08 _____, 0" we get

ds {:l (dr,j2 1112

-='1""+ - r

dfJ ," dfJ

. - - (2)

T11iB is the form.ula for the deriuatioe uJ~ the orr len{:{th s ui.r.t. e

Similarly .. we obtain the ful1u\<v',lng forr]Ju..la for the deriuatioe ol S lV.r.t. r:

., . (:3)

Some consequences

" _ ·t·.- '.' .", . ( r~_ •. -_. ,j~ .. _- ' a;:.:'



Recalling that" tan 'q,

rewri.tten as

ds

- r

de

1 ~ £.. "

1 I cot {p -,- cosec o

- - --

;!; SCF ParL-I. Section 2,:·L l ,

72

ds ~l ',J I L ~
- I tan" ql I - sec <p
dr -
T'hest~ vield
~.
,I r (·0 ~ tl'
-
-
c /...;;
tiD I . ¢
(Is S.Ul
r · . . (f))

· .. l6 J

· .. (7)

( dr )2

fft] +

. ,2,

2[ G.fH 1

r - == 1.

ds )

· . , (8)

and

~ 9 ~ ?

((l~) = (dr)~ + r" (d8)~

· . . (9)

The following Figure exhibits the geoJllp.trical connections among ds. Jr dB an d_ q).

ell'

Figure 1.13

Example,

ds ds

Find d~ and dr [or the following curves .'

(I) r == a (1 - ens 9)

» (j.)

ds rili

.I ~

1 r i

I) ....

l il r ...

, (ill J

, 1,2

t

J

- n 12

1 '2

Il l) o 1

2 ;slH.... .

2

\ ~

e - 20 Sill 2

ti,!.;' fir

1 -I-

1 -I

"

, ....

:---:-1U

I, r'.t.

- 11+----;

I a"

___ ~ __ ll/2 _ 111 + <

(1 cos 8) r (I.'"

~.

-

1/~

I

2n

(ii) From tlu: g-j ven eq u ation, we get

711 ]()g' r == In b l~~r a + log cos In 0 DifTp['(_·niiai1.J1g both srdos, we get

1JJ dr r (it)

In~ sin, nr(:l co~ 17l(-J

Therefore,

or

dr (It)

- r tan InA

2 2 2 _ [1/2

r + r tan cine I

21' +(I

73

- r sec tn 8

1 /

, , rn " . 'I· .

a t cos 711 0 J sec fnH • USIng' Lhe g~l ven ell uauon .



, (In n sec

ds tlr

{ 2 ( dO,:' J2 \1 2

,'l+r -_ ( ~

rlr

oJ

J 1 ,.,
2 nIH I - T~
1 co~
-i - -
- I :2 J -
sm rne Exercises

1 }/.fJ! ( , .0) 'l11.u

1 H](2
1+ ~ 1
r ( ~ . ~ ,
r tan nbS
j 1Jl1 In}2 r2
1+ tr (l,
( m / nr j~
I 1- r a,
~ ~tn ill 2
11 ~. r r
2n1 2,n
(1, - r . ds - 1J

.1" FlTI,d ]_.- Ior the fo .owing curves:

- (,d)

l i) r - -;:= a (1 I co s e)



Ui)

~l C::{'F~ H r --= ae

'1'

(iill r~ a ~ti~

J~r J ds d ds f" 1

~I~ ~ 1-11 - ·"['1 rnr I ,,1H t 'lll~, es :

~~ , .': de Colt cl J' - -,.,

(iJ r - at)

] h d,~;, f']

3., -'rove La l r I 1 ~ consi a nl 'or t re curve a = cos

lr

1 r h

I"j I}

\}k- - r:

r

. J ~I

2 . ( ~ J n V 1 -I 6~ ~

.10 'urvat r

11 r~ 2 '.!r" ~- a

( '2 l/2

(f ' 1 + - ~

r

;;

Consider a curve Ie in the ..t:)'-p~fUle and a point Pi on it. Let \~ he the angl .. which the tangent at P to C makes 'with tho (positive) x-axis. Consider a point Q, _n ear to P~I on. C ~ Let '¥ + S~~ be tho angle which the tangent at Q tl) C makes with the x-axis, A lso, lei & be the arc-length from Pi to Q '

)' ,

o

I

Figure 1.14

Then 6'41 epresen s the change in the ngle \1' curt spending to the change 8.5' in the arc length measured along C ' IG urn I rically, a chs ge in V r !r·e.'rel - .he bending of he curve C"I and the ratio Sl~/o.s represents the ratio ofb -nding uf .. ~ betw en 'th ' poin s P and QI and tl e arc- -ngth betwlu "n them, Therefore, tIle rate of' bending of at P (with respect to the arc ! mgth) lis given by

d~J = lim ~~I lis Q -:. P OS

. , , (1)

This rate is called the r;u ruature uf l]10 curve (.' at thupoi 11 t P. It is uaually denoted by k.' 'llo/JJJ'lI - a l lreek 1 t er). 'I'hus.

. . , '2 ~

rs

II follows that thE curvatu rp or n curve a t a point i~ zern if and on Iy if the curve does nut belld At the point. 'Nute that a ~LT:J,ight line dues not bend at any point. on it. Therefore, the curvnt ure of' Q, ut rcugh I I ine is zero ~ at f~Yt·lry point 011 i L). On the cthur hand, a circlo bends uniformly al C!VG(V point. on it. Therefore, the €·urpulu.re or (I. circle is a cnn ntunt. t. o.

1

K

1 1 (is (,"1 )
P -
(ll'11 / ds} (ltv . . ~
K Remarks: From expressions (2) and CJ) we note that t( and p are both positive or ncg-Htjv(_~ according as (b,~llds i!:>i positiv« or negative. The sign of dwrde determines 1 he convexity or the conrtu.itv 0[1.11(- curve. H'we are n1t)t concerned abou t t.he convexity or the concavity of a curve. "",'e take the absol ute tnumvrcialt ual.ur of d~Jlrls while determining 'K and p , discard il1g the negative sign :if nIl.V. 'I'hus. forul.ulas (2 J H nd ~:3) art' generally employed iJ1 the form ~

1c-

f'Ii. -

p -

l;ilO.l Rarfiu s of crrrv at uz-e in Car-teaiurt for-m

StlPPOS(~ a curve is sppci fied by the Cartesi an. equation Y == l '~r}, Then we have

I'

(/'V dx

tan '~I

And

r)

tl':»

i__ _

, 9 dx ....

" t/~

( 1 + tan':' III ~

'1' d»

. . . { ] )

13 'LIt

{I\J~ ds

1 r

(Is dx

. 2 II ~

{ ( dy J

1+ ~

. I,dx J

1/2

I 1 -t- ()~ ~)2 ... , (:~)

f 3/;) 1
"'~ ? l ~.
J l l""'J -
P
I 8/2
1 + (v ,)21 _
or p - 1 ~,
-
yn' This is the fa mula for 1, radius of curvature at a point (x _y) on a curve y == f' (x) .

(1) The fonTlula (4-) ]s nut useful to find p at a point where the tangent is para U u! to th y-axis; h 'cause at. s uch a p{.li n. y I = tan \1' is nut defined.

(2) ::) ince the I'"tid ius ur eurva ure P"~ ~ a :gf*rnnetrica l pro perty of a CUl"VC: that is i ndepcnd nt of the axes of refer mce, th value of p remains unch~1nged under the change of axes. T'hus.we fUMY change ...Y to x and

;" tu - .-, l w hich U Hl.O untx fo rotati ng . he axes an tiel ckw ise t ,:rnllgh 9n~) in th·· Iormula (4) '0 ~e

1 2 13/2

I I +- ({ix/d)' J 1

( d'.!.xJ dy'l. ) This fhrn1u]a i~ useful for finding p al a point where the tangent 18 parall I] to thev-uxis.

(3) As pointed out '·~n·L~eJ·., p is ChOSIl vIIi .11 a positive sign, Therefor i .. in the Io 'Uru1U1],~, s l4) .a nd (5,l1 th · ,,, l1~O~D ~ v al ules of th righ ,-ha]~d sil ,es a n~

I • - (5)

Ex~ mple 1M, p1i.1,~,(1 the rudius of' curuotu re at the poin t (.x I .v) v:f each . l{ [the tOll I, u i ng curt '-]S .

I • I (.t )

V -.'S Lll ~ ~ . '

1

t:o,sh

77

f ~ l ~~ /2 q I 3/2

P = i 1 + (y) I = c l_1 + airih" (x/c_l..__t_

J) ~, cosh (xl c)

'J 1 ~

J .) 1.)/ ~

_ t: L cosh .... tx/c) J-

cosh xic

c cosh2. x c

2 L

(Ii) Differanuiating he given equation, we gpt 2ay)f"

',:1 2 .)J::

'j 2

• X I), ::J

-- bucuuae (L.Y':" - .1.:

o 1 ,,) · II,. .,. ,

'I. (.1 / .~

ca X (t.,

01'

.' (,' )1/2

3 .l.

2 lI.

Differen tiati n g thia, we get

Therefore,

p

'I'

9 X J'1/2 tl a



4 ~ax 3

.Y

(iii) Differentiating the given equation W~l get

..

or

2 - 1/,:] l 2. r - 1 :3l ((V 1 - {l

3 x 3 ) ,1 J

.' ax

J

((y 1/:3 1 ,I :~

I _ -x V

{,,:t •

I

'Y

L.

Di fferentiating this, we get

/I' {I - 4/3 1/:1 ~ - 1.1:1

V = - -- 3 x .Y Li X

..

........

I

- :~.t

v

,

l'J f).J 'J"

. <..l! r~/'') + _~ d \

I, \ _\, .I

18

1 - 4,'-;j - l/~ ~/;l

3 .::[ .v tJ r llsing the brl.ven equul.iun]

p

:l -,

I 1 1,~·:2 1 ~

1 + 1..)1 ) I

'l-

V

- 2!:l l/,~ 11 ,'3 ~ I~~ 2 / ~1 ~V··~

- 3.(1 _y J! t X I .'V )

I 1 t;j ~./~ 1 l~l o { ) 1. /:~

- a x _v a ;::::: ~j a x'Y



...



Example 21 Sh,OlU that the rorlitts of curual ure or the CU7'f)e

• r _

_ v:::: 4 Sill X - s'in 2y. at x == rc/2 is 5 \j!5 14 .

» From the given equation, we get.

.v ' := 4 cos x - 2 cos 2.1- j

N

.1'

- 4 Sill X + 4 sin ~'t

p ==

3/2 ( 1 + ( 1}2 1

l Y '. ----L.,_I -

n

y

J 1 (4 ~);fl, )~' 13/2

1 + . cos x-,,=, cns z.r ~ J

- 4 Si11 X + 4- siu 2.:y

At the poi nt x =- rr/2.~ this beC0111eS (1 + t!)3!2 -4

(P)X - IT, 2 -

4



Example 3!! Find the radiu« of' curoa .. ture of the curie

'~ 3

..,_. ... i~ V == .3ax~v (1.1 the point (3u/,2 l 30/2)

» Differentiating t].l(_~ g~~V(!l1 equation w.r.t. x~ we gpt

.3·'.:l~ 2 _..L 3 ... '!;',' .•••• 2 .}I ~ ;;::::::

. ,- ~' ~~ a tY' + :~V I)

ay - x2 ')

J)"-' - (I x

, . . (i)

, I) I)

to:v - '2x} lV.... a:\-) - (o.V - .r"'') (2r.v l - a,

- --

• ') I'}

(~ .,_.......J

tr . - ax)

I. Diff"el~f!ntial (}alcu.lus

l\t the point P (30./ ~) 3([/2) ~ (i ) and (ii) become

"I

J'p

-Hn

; , I J. .) ,,;:;;.

1

,} IJ

n ~/4 _ 'J ''''/1) JU I'_) a ,,;,.I

a

p ==

3u 8 \112



Examp-le 4. Fin.r/

2· )

2. a 'a _. x

- - \. at

_y

.x

curve

the point (a 0).

» D ifferen tiating the giver} equation, we ge

:~

2. ~ a

."y-v == --

• • 1) :;;

x..L...I

or"

i a~1

V == ---

~, 2'Vx2



. . . t i )

Evidently. at the point (OJ O}, w" 1 docs not exist. That is, at /t"he point (a, 0) the tangent to the curve is parallel tu the y -axis, 'I'herefore, w l employ the formula (5 ~ to compute: p at this point.

From (i), VlP get

(I,'J

. , ) . . . II [ .

j I.. d~r 1 .c.yx 1'· I u.v

. ,

. __ (iii}

I),

I"'"' - c .X

I)

dv ....

(J

Ther=forc, fit the puiu t (0, 0) ..

7:. U .. El.gille,prin.R .. Muthelllatic!;-11

. ~ I:j .2

1 -t c d x I £1.11 I I

1

a 2

p

-- ----

,) ~ 'J - '} ,

E '-:1 nIp].:! 5. In the el lip«: LX .... /(l .... ) + (v- / 6 .... ) = 1.. ehou: thtit

the radius of .uruature (If. Oil C!Jr! nj'the mujor axis is equal to tlu:

S(~/J1 i -Lo ius rectum.

2x 2'v

-- + 'V I == O.

:2 b~'

n

,

V =

b2,y

? (l .... ·V

.. (i)

This gives

n

,1'

. ~.) ,~ ~,

b ~ ~ (1- -_.V) - ( h .:. x) (, a y ~

.J .)

..:.. '-'

la .'\1)

u.sing (i)

. ' , til)

p -

'Jjf)l

, ,,2 I 'J .~

1 + (v') 1

,}

....

If!

.Y

i ~:2 2 2

h (a Jl I b:» J

j 4- 2: -l I -1,... :!) 1 /2

\(1 J.r , rJ _ ... '

I) ,I' LJ Il ')"

... b" ( c. ~. I b·.... I.~ ~

a. ' f1.V ,,;r J

, . ~ (iii)

A.1 un end of the major axis, V{8. have _x = -I~ a ~ v == O. At such [:1 point, ( iii) becom 08

p

semi-latus rectum.



a

Escercise s

1. Fur pat'] I of the following CHJ'Vt,!S] find l.he radius uf curvature n1 u point rx. y) :

, ~

( ; i i 1 V ~ - 4 £0:

,

tiJ

q

r i v] xy = a J:.

V- r--

t) x "IV 1

i v n + 'Y'h -

2; Find the radius of curval.ure of the curve:

:1 , , ,I .-1;

[ U} n V - X U '- IJ t j h v p n 1 n t ~v he rc i t (TO s S c S t h l2 X -a x i ~ .

(i i i } \,I.t +- .. Ii = ,r(-; ; rl, Lite lJDi III w here .i l cuts the line y = )L'

(iv) y2 LX (:3 - X~) al, the points w~.er~ t.lu- Ln,ugeuts are

pari] Ilel to t.ht~ .r-: Ixi~.

log x

y--- atx-l x

:3 3

~ (1 - x

(vi) v ~-

at the point ~a, 0)

t,

(i J c S(~C (xl c )

2 ,~ 3/'2

tX +)')

( iv)

a: S '/10 a 1 ::d ;j(l
2 CD ( ij) "- (iii 1 (.tv) +v ) ~ vi)
~ a ---.--. -
, . '\/ ~
'J2 a a :~ ;l 1 .. 10JJ2 Radius of curvature in, paramet:ric for'm Suppose the equation of a curve is specified in the Cartesian parametric form x == x (t) and _y =; y (t). t being a parameter. T'hr-n

J [ly avidl-
y - .
'. - -
L dx dxr dt
3 d J l/'ll /(11 t
and I'~ d. v
'V ...... ~
- 2 (Ly 1 clx/ ell
dx
J ,) dx it,)' Lj l
dL") d~x
- -
l Ltf:!. Lit dl ~
tit
( dx f
(It
) , .. ,( ] )

!l_ J ~0!. / t f f l (It d! ! dx / tlt J (ix

T"lI~ E"l: ineerin .. g Ma til t!Jllatic'.t;·Jl

( d 2.1~ / ,e./ f l) - t d." I d f) ( d 2 x/' d l"l. )

.....,...__---=--_.;......

(dXldt):1

l7Hing tl) and r2) in tIll forrnuh n - ll+ CV'J:l ;12/y" \WE

2)

ub .ah

• i')

) 1 + l r.l v I d t J...... } . ·

1 dx /. 1. ,

-

-

I ~ . , , 2 l ;]12

I (dx I ci t ) + ((i~'V I cl t ) [

d t d'.(.} d 2. d d d' I' 2

( .. ~ /. .ll (, ~ I_ t ) - I,' / I l) r( ~-t /c t )

. " " .r8

l'hi ~ i ~ th-.: fnrmulr f-or th· radius of" curvature in the. 'artesian p« rametric fr. 1'""1.

t.~~2 ,+ ~1.~2 ) 3/~ P = ~(' ~lj - ~~~ x

. ot: 'Of many Cartesian curves, the radius of curvaturr may b· obtain d mere (:onVC'I i ntly I ~"'I' using l h param i.ric qu itions.

" " " (4)

~.x,ample - ~ For acb. of' the follolViu{f curues. [uul. tile. radius of curvature ai (1 point ton. the curiu .'

(i ·2 2at (ii) ~ tl 1 l
x - at )P - ].~ ~ ,[I ( + SIr - a - I 0.:..
- " -
3 1
Co ii . .'Y 0 • L t
J( -. a C'os - ~n
- ~ » (i) From the give 11 pararn f!tri c . qua tions Vi g -t

{I ~~ ". d'2~\~

X = ~~ - 2a t' x = . = '2a

(It ~ - '1 2

dt

. (f,

v = ---!........ = '20'

. tit '

" "

v= }' -



'I herefore by formu (;. (,4), 'I e ra I'"II! of Cl rvatur at a .. oint t of the given curve LS

8.1

2' (1- 1,~}:1:!

-= a +

,

on tukin g t,h~ n hso l u to val tLC.

(iiI Fr0111 th(l giVli~ I Pq uat.iuns. we get

.r - u (I I~ t' l)~ f) :V

rr Sill t

.r

o l'u.~.t

Usi ng' these in fornu rla (4)~ VV(, get.

p

~~/~ J 'J ~} I), l I

~ ~ ( ] ~cu~ I)'" _!_ fI - S'i n ' I J

a 2 c O,~ t (1 4- en.'-' I J I a ~ ~ i n 2 l

40' cos tt/2)

(iii) From the ,brl,v'on equutions, \N'{] get

. 3 2 ~. . 3 · '2 ,J. '

X - - -0' cos t sin II 1 ,'\: = (l SU1' 1:, C'OS t

~ 3

- 3rt ( - 2 ,r us t s in ~ t + cos" I)

x

, , >j - ')

~ - 3(( C n~ t 'COK~ L - ~ S t n - l)

~ ;j

31(1 (:2 sin t cos' t - sill l)

v

+

, l) • i)

:30 sin t ~ 2 cos .... t - sin" t)

Ls~,ng' the~.H.!~ we gf~·

'J ~J .,.

2'7" ...... ' " ,]I

;;::;:: ,- a CO,S I. s 1"11 I

The re fore ", u~rng formulu (4~" \VC g '1. 'J 7:l '~/' a _ " a ,-"os ' ~ U l f

p

'..,I • J ')

~'(,/~ ~rnu~'1 ('(pg- I

= ;j(J si n t. cos I !



"Example 2.. Prone that the radiu.s of' en routure at; a point ofth»

. 2 IJ)I ~ I? '} 2 3;

ellipee (J..- In""") -t (y Ib' .... ':; 1 18 If/veu hJl p == ,(I .... /1, .' pH l' uiherc

,

p is the lengt], of I he P" rpendic u./.o r [rom the cen tre onto t lie

tmuien! at thrtt point

J: '= {I cos t, "I == b si n. t.

The~c give

. X ~ - ,a si.n. t. ,

.

y - b cos t ,

u •

x == -'(J cos. t

, ~ · 'l ~~ /~

(x -1- Y J

p === • I~

J('V- :yx

2 . :l I) :2 - 31'2

_ (u,' SI11 . f -1- b" CDS _i1_

ab

.. \

.. . .. tn ~

We note that the equat.ion of t]l~l tangent to th.e given ellipse at

th . t '~.' .r cos t. ~~:s in" ITh t': tl iI eth f tl

' IV pOln. ,~: IS -I . - 1,' tJ;I .. 't·~-ore, l'l,C .IIlCJl:l,6'; .' rn . _le

a. ~)

perpendicular from the centre of the ellipse (the origin) onto this tangent is

1

. ( ab )~

p== -

p

This proves the required r(~sult.

Exere isee

nb

2 . ~ 2 2 1/2

tiel SIn t -+ !J cos t)

1 ah



Fur each of the foUowing C-LU\1t?~~ find tho radius of curvut.ure at a point i :

ti) x = n U - sin ,Ol )' ~ a (]1. cos I)

ti i ~ .r !: a (co s t +- t ~ 1. n I ) j .'!,I = a \ si n t - 1 cos t)

~ ~ i i) x :;;- a. In g (sec t + IL a n I) ,. ~ iv ) x:;:; (1 [ CO s t 'l I og tu n t /1:/),

~~ - a sin t

..

1. Differential Calcuius 85

( v) x = 30 co s i-a eo ~ 3l ~ y.=: 3n sin t - Q sl n ;] to

( v i J x == a 8 in 2 { (1 + cos 2t), Y == (J. cos 2 t II - cos 2 t )

Ar-ISWerS

(ii) a!

~ v) 3a sin t

2 (iii) 0, set l'

(iv) (L cot I

(vi) 40 cos :3r.

1.1'0.3 Radius of curva.ture in pedal form

Suppose t]1C equation of a curve is given in the pedal form. Let us recal l that lV = 8 + ¢I . ~I; This gives

1 ,dllJ dA dq, de dth dr

_ ...::::....I. :=: _+ _ = + ~ _

P ds d s .. d.s ds dr ds

1 (si n~ .m d _.) . ? ti\ de d r ~- -'.

I S 1 n ~ + reo s~; d •• ~ uSing 811] '¥ ~ r - ~ cos ~ = -{ ,··r·

r ir ds . CS

1 d . 1 d1! .

~ r dr (r SIn tP) ~ -;: ;:. ~ using p = r sin q, *-=I=:.J:

dr r-

dp

'I'his is the formula for the radius of curvature in the pedal tp-r] form.

p =-

... (1)

,A consequence

As a conseq uence of the form ula ll), we m.ay obtain H forrnul a for p in ter111S of p and q:t.

Since p :=: ,ds/d~J

formula (1.) may be rewritten as

ds d\V

dr ds d's

- r - - -= r cos ~

ds dp dp

This vields

...

r cos cp

. . - (2,

Also, we have

------

.,~~:! See S,{'ction 1.9.2.

SeQ Part - L Rt~t"lio u 2.3. 1

86

J} ~ r sin. ~)

.... (:3)

Squaring and adding (~) and (.3), we get

.~-

[ ~~ J + p2 - r2

Differentiating thl~ W.J" .t. \~, we get

dp 'J !lE_ d,.
:J ,d_~J! 2jJ 2r
+ - -
.iO.oI d\lf .' 2 d,V - dql
d1Jf or

d2,

.P + p

dt~/2 .

Using (1), this becomes

dr

r rr= :

d'~,

d14' _ dp

dr r-,

lip

d2:

P·· = p . ...!I- -p

'I IJ.

(lll/~

This is the formu.la fur p in terms of p and \~; t.his is referred to as the formula for p in the tangeniiol polar form.

. . . (t1)

Example. Find the radii of curvature ofthe ,{ullolVillg cuues:

. :2

( .. " . 1 1 1 r

II) 2:= 2 ~- ~ - 2:2

I) a b ab

» (i) D iffercn t iating the given eq u a tiun w.r . .t. r we get

-. gp_ _ .. _2

4up . - a, OJ~

. dr '

dr _ 4a,p dp 3r2

TI l·"~(." . ·b· '. I:_ . . 1· I '1) ". crp·t

.It rcrore, ... y !ornlU a \ - ." we b~

3/~

_dr 40£ 4a,.'~

P _. r -- == - ::::: - - ~ DTI using the given equation,

dp ~ir .3,. ,i:2a

2 1'--

12' -

- 3 \(Jr

(ii) Differentiating the given equ atirm. we get

dp dr I). L)
- ',), ~ 21" u"'b~
o!...J
- or - -
- - ~,
:~ tlr . ~ .:1 ~ r/j]
1J' a ,Ii p'r 1. Differ'e",t-ial Cnlc:u Ius

'I'hereforc ..

87

'2.b2 . f2
dr [ 1 ~ 1 2
a . 2 I) r
p r- - a b" -_
- ~- -
dp 3 2 :2 a2 b2
!) a b
I , 2 :l/2
0-+ b~ - r
n.b •

l;'~nd the radii of curv ature ~. f'Lhe fol1ow~ .. n,g curves :

2 3

(1) pn ~ r

('_.1 n il"-l 1

rot pC1 r

An. wer ..

"t (1) a 3r

~ (2) pr = a

(3' 2

] P := ar

(~). 2

i P ==

r'~

'2 2:

r -I- (I

1.10~4 Radius of [u.rvatl r'e in. p1ol,ar' form

Suppose the equation of a curve is given in the polar form r ~ f(el). Th In w'!? have"

2

] (11: )'

2+ .4 d:"

r r,ij,

. . . (1)

Differentiating this w.r .t. f, we get

2

- 1. - 4 (dr J - ~

,."1 ,.~l. del. ,..4

'2

,~~ ,.3 2 ( ::~ )

1 djJ
or ~
3 -
dr
P' Therefore.

2
:1 1 •
~ 1 dp e. 2 'J ( l~ d~l~
- I r I'" ~
P r dr A ~ de IH~
r f_. 88

T.lI. Ent:itleering Matiu:!lnatic8-U

1

6 r

or p -

6 r

- -

,) .

':;I - . .... ,} '} •

r~ + 2 (dr/tit)) - r d'r» fl0~

by us in.g ( 1 }

, [) z l,3/!]

i r .... + ~ dtl de)i

? 222

r" +.2 (dr/dO) - r td ride l

This Ll-i the [orm ula tor p ill the polar form.

... (2)

Pu t.ting r 1 ~ dr/de

2 .2

and r 2 == d tide

may be rewritten in the following compact form: (r2 + r~)a/2

p

'3i

, ~ .()

-

r2 + 2r~ - rr 2.

Remar'k : For some curves, computation uf p by using formula (2) i.~ a tedious t.ask. r n such cases, p is enrnputed alter reducing the polar equation of the curve to the pedal form ; ~et! Example 2 below,

Example 1. Find the radii of curvature ofthe [ollotoiru; curves:

('''.)''1 Ii· u II r -= a Sill, n II

» (i) Differentiating the given equation w.r. t. e twice, we get

dr

f1 == -,- - - a sin e l!

' c l~

(1"21"

r ~ r == - a cos B

2 df)2

These give

2 ~ 2 .,."). 2. 2 0)

r + r = a' II + cos (1)- + n SUI tJ - 2(1 .... ~ ] CUH fH

. 1

2 '1 e 2. i) 1. . 2 S 2 u 1 tJ'

- (J t + COS·) I .at 81 n H I a eOH u ( I cos " )

')

- 3rt.... (1 -I- co s U)

p==

'{ 'J ,~

2 \,' ~ a . ( I + (~OS A), I ....

-

')

!_l,(l' - ~. J + cos H)

1 'I

U. (1 + (~O s Ii ,)

., "

r" + 2r"-" rr

. 1 2

89

Note: Observe that r2 ~ (8/9) nr J or p2/t - tH/9)a = consumt . (ii) Taking the logaritl1111 of t11~ gi ven equation, we gpt

/1, lug r -= n ]()g a -f-- log- sin ne

Differentiating this w.r. t. A j we get.

n dr r ciS

n. cos nA sin ,r~8

or

clr r1 - de

r cot n8

Tbis gives

2

r 1 tot n e + r ( ~ 11: C osec Ii,a)

2 .,

r (cot n,8 - n cosec" n.B)

Therefore,

2 2 ? - ') 9 2

r + r 1 - r" (1 + cot" ne) :::; r ..... cosec nH j

2 2 2 2 (1 L '2' t2 n t2 £j 2 O)

r +":11 - Tl" -- r' r CD Il ,,- co n" + n cosec n n

1 2,

2 ~

= r' t n ,+ 1) cos ec .... nS

The formula (,3) IIOW gives

U,,2 2 . 11/2 1 n
cosec nfJ) r r a
p - ~ ~ -
- - -
2 ( cosec2 1 - nH L 1/
r n 1) l1H n + sin ri + r n a

1. '

using tho g1ve'u eq'uat.iort.



Example 2~ [find the radii of cur uo lure ofthe [ollounng curves :

(i) r == ae 0 cot ~t

q OJ

( ,~ • ) L. 29 ....

[1 "COS"" ~ (I

», Here we find p by fLrst red ucing tIle ~ri,,-pn equ atinn to tho pedal form.

(i) For the givpn ClJrVCl \4{ ~ have

dr. H Co I, II

-,- = tfl en l cO e - r coL It!

elB

lfA t an ~~ -= r

dr

- .,au I~t~ ~

.'

so thut t~ == ll. Henes-

.

r 8111. 01 .

'I'h i sis t. h t" ]J ~d a I C!' q U [It i l in 0 f tl 1 C ,gi v ell r UJ'" V [~ I Fro rn this cq ua 1 ion, we g-et

r

= r eusec c( .

.

stn U

,til) dr

1

eat 2B.

r

T]l C'rl(~fore1

dB f 11 )

I an cp = r til" = cot 20 - tan l 2 - ~H

(1t/2) 20. Hence

IJ r silt lID == r sin ( rr/2 - 2tA.) = r cos ~Q

I) ~
(I .... a
r • ~
~ ~
r r on using the g-iv~n equat.inn.

Thi S i~, the peda I e( 1 u ati nn of th p gi v~n c u rve Fro U1 this ~ we get

df!_ .j dr ',1'
(1- r"
- - or
dr 2 ~ lip ~ .
r () Tho refo re,

3

dr r"'

I) == r ,j--'= '" '2, '!!

{J~p a



I. Fi n d th p radii or ,! urvatu re uf t h [~ fb llnw j,ng' curves:

.9'1

2.. Show that for the curve r = a sin i~~H ~ th.e radius of'curvature ill tJu= pole is .nn',·2

3,. Sh,QW that the radius of curvature at any poi nt on the lem niscate 2

r ~ o cos '20 is ~11 versely proportaonalto I' .

1. (i) ~

(v)

11 1 (J~ +- 1) r

(iv l ;lr

1.10i5 Cerrtreof c'urvature

Consider a curve C in the x.y~p]ane an d let P be a point on it.

Also, let p be the radius or curvature of C at P ~ ,COlu8id,er the normal to C atP and cut-off' th- length PE == p along the normal. Then the' circle whose centre is E nnd radius equal to Pi is called the circle of curoature for the curve C at the point p ~ The centre of this circle, uarnoly EJ is called the centre afcurvature of th,p curve C at the point P:

J" , T

o

c

M N

If IQ is a ncighbo u ri ng point o f P on (.' ~ then t~1P (I. rc J?Q m a y' be regarded as an arc of the· circle of curvature at P ~ Hence the normals to C at P and Q meet at E. Thus, the centre of curvature may be regarded as the point uf intersection of normals at neighbou t'"j ng; points.

Below we d'crive forrnulas f(n~ 'll~r ro,(n~d.inat.0~, (x" .1') of the point E ~ We considCt~ two cases.

('lI,~ ."'~

Druw t.hp perpondiculurs PlY and En1 on . he .r-axis, and the pr-rpoud icu lar I'H nnto EAII; see 1~i6'1Jrf' 1', 15. 'I'hen, from the li"_g11re~ \\!P Crnd that

-

.t

x ~ r sin ~,

LV - ;1!1 'f - NIl + R'E =mrJ -t EP COB ~

. , , (1 ~

.v + P COB ~I FI'Onl an \1I := .' f it loUo\\rs .ha

• I' (~J

..

~lI1 ~I :=

~~l

:y ~

-. -J--'lj_

tv r

and

]

l\Jso,

.

p =

-

,1/ := v

~ , t (3)

T hcse aro the Ii )T711lli {J,~ [or tlu: coortlin n te« of' I he ce ni re of en rua t ure in ,Ii c (:i(J I' I tsi an [orm ..

Case (i i]: Cerrtr " of Cl]r"Va·ture in par-ametr-ic fo·rm

Using those in (J) and (2)~ Vile obtain

• I •. ')' .' ~ 1

- l' ~ (.'l:) ~ + ()' J' .

,.\~ _ .. \:' - if::! .... ... ..I'" II. -.. r~

X .V - .\~ X

-

,v

,J ' ,2 <I [";, I

p x I (){)_.±_ v)'" J

\ + -.... .,.~

~ _"t' .V - .Y -r

. , ~ ( )

These are the for-mulas ror tlu- coordinutes ur the centre oj' curvature in the ICJ(Jrt~.~·,i~nH parnmetrieIorru,

Eql"utiul', of til (.1 ci,r(:I~'1Jl ut' cu.roatu re

\~'ll (~11 t~][e CU~ rrd i n atus ~ .. r ~ .\7, uf the cen rot' n r curv a tu rc at a pOilll 1) on (T :irf~ d(-dpt'Hlln(_~d by usiug 111[': turrnul as (8) 1]01' 4)! he . Il~1IJ at.ThI! 11 of th p ei r·t.JI ' of" 11,,",; tu r ~ ~ t I' ~ S },vi''"CH by

Example 1", Find llu: centre or ruruulu re 0 f, (I poin i (Xi .Y) ().( "he calcnar v .1' ;:::; U cosh (x/a.).

» From the given equation, we get

Thercfbrc ~)V !~Of"U:dU L, S (~3', t hi;' coord inat s (.\~ .. y) )1' thp ccn re ill' curvature arc

_:r

slnlllJ (x/a) l' ~~nh"l (x/a

_ - ~_~

(11 (I) cosh ~xla}

- .\"

;z

,.-;; i 11 h tx / (I J ."1 H·d J (:r In)

r_, ~

. ·CO~"5 h (.t I (l' )

J"' - o sjnh (x ) cosh (.f J

(I , (JI

II

V

..

Example 2.. /( ;111/ ih .. centre utu] the circle of CUl"l)(I i lire at tlu:

.-

IJoinl (0/4 I (1/ 11 oJ'th(~ curve v.\~ \~V = 'ill "

I ~

,~ 4

(_v ) l:J' =

r (I

The- rcfore, tJ1C' C. iUI'~d _i n rrt('~ (.'I: I J' Y I)) nf tlu ("( lut rt1 0 j' ell rv a tu 1"(..1 ~ ~ t P r~ rt: I by rt U' In 11 LIs (a 'I.

I ';J l
( ! ) i t ~ { - ] .f·· I
(1 U ~~a,
- - - - - ',- - - -
- ~
Jlu 4 9 4
.:..I "J

1 +. {, - 1 r~ :~o

--- -_,.

4/0 4 .,

AJs ... h ~ ra ~UH 0 . curv: ture at _ is

:?i ,,"~'.

,........

\12 ·

OJ

Hence, he- eq uatirm of lh~, circle of curva D.r a IJ ~s

.

t.e.

( 3n)2 (. 3u J2 tJ 2

;'t'- - r Y-- -

4 ~ 4 2

which simplifies to

8 (x2 + y2) ~ 120 (X + y) + 5a2 == 0

Ex,am'ple 3, Obtoin lh ~ coordinates t~fih ' c nlre of curoature at

? 2

onJI point (at ... ; 2.af) of the parabola .. v·· = 4{L'r (l.,nd prooe that the

. f' l . 1 'f I

equation 0 the eire e 0'- curvature IS

2 _ 2 '2 . ,:~ .' 2 .,4 a

x + }- r6flt x - 4u.x +at ,,- ,i,ia' t == .

~ ~

Therefore

1

f- d)f d» /tll 20 1
'}' - - ~- - -
- dx - d~tldl 2ut -
.. t llb(f:::;e giv _. the courdinatos (x ~ .. y) of the c nt fP of cu rvaturc as _T = al' _ ('I~f Ll_1.:!_ 1{!_2) = (I (2 + :3t:!)

J I (~lOI:l ~

'1 - Zat' .

,UIO:;::

Ik:l • .!II

Also, the rarlius of' eurvaturt: is

(1 -1- ] I t2 l;3 ! 2

p:::; - ~ 2u {11 (~)~~/2,~

~ 1/2(1£3

discard! !l,g tho nugati V(~ ~lgn

I ~ '2 :J 12. ~ - .) '-tt

1 _y U (~ I :~t ) I +- I ~y -l- 2n.l I ;::; 4(1 (1 + 1,-- f.

This ~1 mplifies to

'J') 6 - I, 4- ~~ 'I' 4

X- +- Yo) .... - 4a.:l~ - at .... x.... ('Il'y - 3D,:";!

o.



I~ Find the (~entrc of curvature t:lt a point tXt:Y) for each nf'Lhe fol1ovvi.ng CUTV~fl;':

(i t.~ 'l - 4flJ:

~1 :2

(iv: Y - a ·x

(vi) x - (1 (cos f. ~I t sin t, i Y = f1 ~ Bin t - l cos 'l

2:~ Find .he centre uf curval ure at the' given puint for each uf the ff)ll owing' curves:

"j :I

(.iH. J _,t + .'\." _! :3axJ! at (~a 12~. ·3a/2: 1 ~ i v))' - ~,o~ s(!(: _'[ at (n/,3l log- :l)

3.. Find the. circle of curvature at the given point, of each nf i.he following curves:

• j

( i v) .'.1 - = 4ux at ( (1 ~ ~(f.)

AllSW€l""S

_ "'~"'r I..,., .\-. [I! " LJ

(il] x -

OJ ,_II

.r

1~ - 2(1 +

~J

:J.y

4n

,.}

... ) a:

{HI· .r -

oj .J

b" - 0""

t. .~

: ~

\1

.r

f·'

\lVI l"

y -

·1 I

96'

T. [T~ EugIneering .Mntheu, atics-Il

{ v ) x -:::;- a U I sin t) ~

.;.' - ,(

C1 {I - c( ) ~ l ~

( ii ) ~ 5/2, 51 ~ )

riv 1 (ttl;] -\J'~3, 1 + log 2)

(~ ( _. _ \ 0 ... , 7 ') 8 7 ') 'J.'

J 11 J (cu: - .~ .... + t'y - ,)""' .s: c:

,) 2

3., (1) X~ _!_ V I- "( - :~~l' + 2 _

<") ,) 2 2 2

I i.i i J x'_ I v~ I Gx 5 y I 8 = U t i v ) x +.Y - 1 Uax -I- 4ay :3a - D

l!lIO.6 Evohrtes

Given a curve C Inthe x.y-plflne) suppose we find "the centro nf C\1 rvature at all arbitrary point of C. 'I'he locus this centre of curvature is a curve C j in the .xy-pla.nc. TIle curve C' is called. tile euolute of the CUTV'e· C. Further, the curve C is called an in. uolute of C ~' .

Thus, the evolute C I' ofa curve C is the locus of the centre of curuature of C; and C is an, in vol ut.e of C I •

2 Example 11 ShO(1) that the euolute of· the parabola, y = 4ax if'

4(" n ..... 3 '2·7· 2

x - ..::.rL~) == ... , ay - .

I

For 'the given parabola, the parametric equations are

2 . ..

x == at ~ y == Zai . Using these oquabion s, we find that the

coordinates of the centre of curvature at any point t are

»

X' == a (2 + 3t2) , see Example I) of Section 1.10.5+ From (i), we get

11 ,== - 2at~1

...

. . . (i)

(- 2a).'3

x ~

or

3 .. - 2

,4 (i - 20) = 27a tv)

Thus I the t{ xirdin a tes (:r l y"l of t.hc centre o f c urv a t ure (of tIH': given para bol a) s ati ~f}t th P. pql.la ti on

,,;.

.. ~ ?

4 ( _ q ,),1 - 2··7 ........

. \. - ~a· . - ,uy

., ,~jjl

97

'I'his is tJ18 equation of Lht- IOCllS of (:- )'} ~ In other words, this

is the equation ofthe evolute of the givEn purubula. •

tt (co~ t+ t 81TI t)

v == a (si n t - t eo ~ t}

is o. circle.

» Froru the gi ven pnr.ametric equations t where i is the

parm ptl~r), \VP. g ct

J

d)'ldi. - t. . .

d .. d - an L xli

.Y

riv ~ dt
s «
_y ~,
• r -
dl dx ,) ( 1 J 1

... I -

sec .~ =-

l at co:,; f at cnr-:''l I

Therefore, tho coordinates of the cen t.re of curv atu re are

-

'\+ - J't.-

<)

_ ~V f. (1 ~ tv ' ,J'"""'} • I: ~ ,. 2

x-- ~~' - u (cost +ts111t)-ar cos l lan[ser I y

a cos t

== a sin t

. - :t. -- "} 2 Tl ~ I

The 50 gi Vp x . + y -'-' ~ Q _ ~ . 1 U S ~ t h to locu S 0 f (x , ~ V) is t U:

- 2 ~) 2

circle -'r + .y .... == a This circle is thu evo}utQ of the gjven curve .



Example :1. Find the euol uir: .or tl. e cllips«: il = b Sill fl .

.. '

x

» 11~re, tilt-" equation of" '11,0 ellipse is bri,vcn in the para metr ic

form with 8 8R tile parameter. \1.1 u find that

.r r =

d,Y _ ((v/dA .t. dx /dO

a

L) dB b 1
d-y (Iv r l( ,I , I ]
_ I I
'v',. - eOflt'r .0
,) d\ dO dx -
" ~IU e
lhr'" rt I H
~ )
,~

You might also like